Você está na página 1de 72

TEST - 7

User Name :

Sunanthaa

Total Marks :

200

Mark Scored :

84.67

H
TS

IA
S

1 Recently organizations representing the global shipping and oil industry revised the
Piracy High Risk Area (HRA) and shifted its eastern boundary away from the western
coastline of India to 65 East longitude. What implications will it have?
1. Payment of Additional War Risk Premium (AWRP) over the extended HRA,which
added to the cost of transportation in trade, will be removed.
2. The shipping traffic would move slightly away from the Indian coast which will
help avoid security related incidents of firing on boats.
Which of the above is/are correct?
A. 1 only
B. 2 only
C. Both 1 and 2
D. None
User Answer :
Correct Answer : C
Answer Justification :
Justification:India had been seeking revision of the HRA back to 65 degree East
longitude as continuation of the extended HRA had both economic and security
implications. Payment of Additional War Risk Premium (AWRP) over the extended
HRA added to the cost of transportation. So, 1 is correct.

The shipping traffic had moved closer to the Indian coast and the heightened alert
had resulted in unfortunate incidents of firing on Indian fishing boats mistaken as
pirate skiffs. Since the line is revised, 2 will be correct.

SI

Learning:Revision of the HRA will ensure an optimal utilisation of international


resources to combat maritime piracy by focusing international efforts on the actual
areas of high-risk, it said.

IN

Containment of maritime piracy off the coast of Somalia is an example of successful


international collaboration in the area of maritime security.
Q
Source:http://www.business-standard.com/article/news-ians/india-welcomes-revisio
n-of-piracy-high-risk-area-115100900673_1.html
http://www.mea.gov.in/press-releases.htm?dtl/25904/Revision+of+the+Piracy+High
+Risk+Area

2 Which of the following is/are the similarities between the Asian Infrastructure
Investment Bank (AIIB) led by China and the Asian Development Bank (ADB)?
1. Both banks have regional (Asian) as well as non-regional (non-Asian) members.

(C) Insights Active Learning. | All rights reserved.

www.insightsias.com

TEST - 7

User Name :

Sunanthaa

Total Marks

200

Mark Scored

84.67

IA
S

2. The USA has the same voting rights in both ADB and AIIB.
3. Both contribute to the assets of World Bank to supplement its developmental efforts.
Select the correct answer using the codes below:
A. 1 only
B. 2 and 3 only
C. 1 and 2 only
D. All of the above
User Answer : A
Correct Answer : A
Answer Justification :

H
TS

Justification:Statement 1: Both the ADB (formed in 1966) and AIIB (formed in


2015) are responses by Asias leading economic power. Both banks have regional
(Asian) as well as non-regional (non-Asian) members. But, in both AIIB and ADB,
non-regional members have lower voting rights than the size of their economies
suggest.
Statement 2: The US has the same voting weight and shareholding in the ADB as its
founder, Japan. On the other hand, in AIIB, Asian nations have majority
shareholding. US and Canada have not even joined AIIB as they would have got
minority shareholding. So, 2 is incorrect.

Statement 3:AIIB was setup with a view to reform world economic goveranance,
and analysts point out that it will be a close rival to the Bretton woods twins, i.e.
WB and IMF. So, 3 is wrong.

SI

Q Source:In news for past some time.

IN

3 Which of the following functions is correct about the Prime Minister's Office (PMO)?
1. The Cabinet Secretariat is a subordinate office under the PMO.
2. The PMO provides secretarial assistance to the Prime Minister.
3. It generally handles all activities not assigned to any particular department.
Select the correct answer using the codes below:
A. 1 and 3 only
B. 2 and 3 only
C. 2 only
D. 1 and 2 only
User Answer : B
Correct Answer : B
Answer Justification :

(C) Insights Active Learning. | All rights reserved.

www.insightsias.com

TEST - 7

User Name :

Sunanthaa

Total Marks :

200

Mark Scored :

84.67

Justification:Statement 1: Cabinet Secretariat is not a subordinate office under the


PMO, as it does not report to the PMO. Moreover, Cabinet Secretariat was
established before PMO. So, 1 is incorrect.
About other statements see below.

IA
S

Learning:The Prime Minister's Office (PMO) consists of the immediate staff of the
Prime Minister of India, as well as multiple levels of support staff reporting to the
Prime Minister.

H
TS

The PMO provides secretarial assistance to the Prime Minister - virtual 'think
tank' of the PM.
It is headed by the Principal Secretary to the Prime Minister.
The PMO includes the anti-corruption unit and the public wing dealing with
grievances.
The office houses the Prime Minister and few selected officers of Indian Civil
Service who work with him to manage and coordinate government and his
office. /li>
The Prime Minister through his office coordinates with all ministers in the
central union cabinet, minister of independent charges and governors and
ministers of state government.
Monitors the actions taken by the PM in earlier meetings.
It generally handles all activities not assigned to any particular department.
Its importance increases wrt to cabinet too as PM is the coordinator of the
Cabinet.

IN

SI

Q Source:Frequently in news. There has been an apprehension lately that all the
executive power of the Government of India is getting concentrated in the PMO, and
that the Prime Minister is dictated by a handful of officers.

4 Which of the following features of the Indian constitution are borrowed from the French
Constitution?
1. The Idea of a federal polity
2. Directive Principles of State Policy
3. The Idea of Rule of Law
4. Principles of liberty, equality and fraternity
Select the correct answer using the codes below:
A. 1, 2 and 4 only
B. 2 and 3 only
C. 4 only

(C) Insights Active Learning. | All rights reserved.

www.insightsias.com

TEST - 7

User Name :

Sunanthaa

Total Marks

200

Mark Scored

84.67

D. None of the above


User Answer : C
Correct Answer : C
Answer Justification :

DPSP was borrowed from Ireland. So, 2 is wrong.


Learning:From U.K.

IA
S

Justification:The Idea of a federal polity was borrowed from Canada. So, Statement
1 is incorrect.

From U.S.A.

H
TS

Nominal Head President (like Queen)


Cabinet System of Ministers
Post of PM
Parliamentary Type of Govt.
Bicameral Parliament
Lower House more powerful
Council of Ministers responsible to Lowe House
Speaker in Lok Sabha

IN

SI

Written Constitution
Executive head of state known as President and his being the Supreme
Commander of the Armed Forces
Vice- President as the ex-officio Chairman of Rajya Sabha
Fundamental Rights
Supreme Court
Provision of States
Independence of Judiciary and judicial review
Preamble
Removal of Supreme court and High court Judges

Q Source:11th NCERT: Indian Constitution

5 Which of the following rights have been expanded by the Supreme Court in the years
since our Independence?
1. Right to Freedom of Speech
2. Right to Public Offices
3. Right to Life

(C) Insights Active Learning. | All rights reserved.

www.insightsias.com

TEST - 7

User Name :

Sunanthaa

Total Marks :

200

Mark Scored :

84.67

IA
S

Select the correct answer using the codes below:


A. 1 and 2 only
B. 2 and 3 only
C. 1 and 3 only
D. 1, 2 and 3
User Answer : C
Correct Answer : C
Answer Justification :

Justification:Right against Censorship of Press; freedom of advertisement etc. have


been added by the SC. So, 1 is correct.
There is no right to public office in India. So, 2 is incorrect.

H
TS

Right to life now includes right to a clean environment, privacy etc. So, 3 is correct.
Q Source:Page 30 onwards: Chapter 2:11th NCERT: Indian Constitution

IN

SI

6 What is an Eco-sensitive Zone (ESZ) as notified by the Ministry of Environment and


Forests (MoEF) frequently?
A. It is an ecological hotspot which is internationally recognized as a part of
protected areas network
B. It is a zone of high endemism of species
C. It is the Core zone inside a protected area where no activities are allowed
D. It is the area around a national park or sanctuary where developmental
activities are regulated
User Answer : A
Correct Answer : D
Answer Justification :
Learning:As per MOEF 2011 guidelines, it is the area around a national park or
sanctuary upto 10 kms where developmental activities are regulated (not prohibited)
to:
Minimize impact of ecologically harmful developmental activities (e.g.
mining) on fragile ecosystem encompassing the area
Protects biodiversity by avoiding fragmentation of habitat
Act as a transition zone between areas of high protection (inside) and low
protection outside
Act as a "shock absorber" for the protected areas

(C) Insights Active Learning. | All rights reserved.

www.insightsias.com

TEST - 7

User Name :

Sunanthaa

Total Marks

200

Mark Scored

84.67

Q Source:http://envfor.nic.in/content/esz-notifications

H
TS

IA
S

7 Consider the following statements.


1. Assertion (A): The Parliament is bound by the constitution of India in guiding its
actions.
2. Reason (R): The business proceedings of the Parliament are conducted as mentioned
in the rules within the constitution.
In the context of the above, which of these is correct?
A. A is correct, and R is an appropriate explanation of A.
B. A is correct, but R is not an appropriate explanation of A.
C. A is correct, but R is incorrect.
D. A is incorrect but R is correct.
User Answer : C
Correct Answer : C
Answer Justification :
Justification:the Parliament has been established by the constitution of India giving
its structure, functions, powers, responsibilities etc. So, A is correct.

The business proceedings of the Parliament are conducted as mentioned in the


Parliament rule books, conventions etc. The constitution does not detail out the
business procedure of Parliament. So, R is incorrect.
Q Source:Chapter 5:11th NCERT: Indian Constitution

IN

SI

8 Protection against forced or bonded labour is a


A. Legal right
B. Constitutional Right
C. Fundamental Right
D. Right given by executive order
User Answer : C
Correct Answer : C
Answer Justification :
Justification:A legal right is one that is mentioned in the statue book, i.e. provided
by law. So, (a) is incorrect.
A constitutional right is mentioned in the constitution of India, e.g. right to vote,
which is not a fundamental right, but still a constitutional right. So, (b) is also
incorrect.

(C) Insights Active Learning. | All rights reserved.

www.insightsias.com

TEST - 7

User Name :

Sunanthaa

Total Marks :

200

Mark Scored :

84.67

Learning:It comes under Right against exploitation which also includes


Prohibition of forced labour;
Prohibition of employment of children in hazardous jobs

IA
S

Q Source:Page 31:Chapter 2:11th NCERT: Indian Constitution

H
TS

9 Recently a lot of concern has been raised over the software


A. It can be used to change the IMEI number of the phone after connecting it to
a computer
B. It can block the firewall of public computers.
C. It can steal sensitive data from high end computing devices installed in the
research institutes in India.
D. It can suppress the IP address of a computer thus posing a threat to cyber
security.
User Answer : C
Correct Answer : A
Answer Justification :
Learning:It is software which is used to change the IMEI number of the phone after
connecting it to a computer.

The software provides the option of allotting one IMEI number to multiple phones, a
technique that misguides police during investigation.

SI

Q
Source:http://www.thehindu.com/news/national/other-states/flasher-software-ties-c
ops-in-knots/article7649775.ece

IN

10 The Directive Principles of State Policy (DPSP) providefor


1. Fundamental principles of governance
2. Constitutional rights and legal entitlements of marginalized sections of society
Which of the above is/are correct?
A. 1 only
B. 2 only
C. Both 1 and 2
D. None
User Answer : A
Correct Answer : A
Answer Justification :

(C) Insights Active Learning. | All rights reserved.

www.insightsias.com

TEST - 7

User Name :

Sunanthaa

Total Marks

200

Mark Scored

84.67

Justification:The constitution itself mentions that Directive Principles of State


Policy (DPSP) are fundamental to the governance of India, as they provides for the
essential principles like equity, fairness, welfare orientation etc. So, 1 is correct.

IA
S

2 will be incorrect because DPSP only gives guidance to the state in the formation of
policies. The rights and entitlements can only be provided for in a law enacted by
the legislature which gives force to the provisions of DPSP.
Q Source:Page 43:Chapter 2:11th NCERT: Indian Constitution

H
TS

11 Consider the following statements.


1. Assertion (A): Parliament cannot enact a law which is inconsistent with any
Directive Principle mentioned under Part IV of the constitution.
2. Reason (R): It is based on the Doctrine of excess jurisprudence as given by the
Supreme Court in the Ramnath Goenka v/s Union of India Case, 1962.
In the context of the above, which of these is correct?
A. A is correct, and R is an appropriate explanation of A.
B. A is correct, but R is not an appropriate explanation of A.
C. A is correct, but R is incorrect.
D. Both A and R are incorrect.
User Answer :
Correct Answer : D
Answer Justification :

SI

Justification:Assertion: Parliament can enact a law which is inconsistent with a


DPSP as DPSP is not justiciable. However, SC may use DPSPs to validate a law
enacted by the Parliament which violates the FRs.

IN

Reason: There is no doctrine of excess jurisprudence.


Q Source:Chapter 5:11th NCERT: Indian Constitution

12 What is the significance of the Turkmenistan-Afghanistan-Pakistan-India (TAPI) gas


pipeline project for India?
1. It will allow India to export natural gas to the Central Asian nations hassle-free.
2. Indian government has also announced extending the pipeline to China and
Myanmar to diversify its markets.
Which of the above is/are correct?
A. 1 only
B. 2 only

(C) Insights Active Learning. | All rights reserved.

www.insightsias.com

TEST - 7

User Name :

Sunanthaa

Total Marks :

200

Mark Scored :

84.67

C. Both 1 and 2
D. None
User Answer :
Correct Answer : D
Answer Justification :

IA
S

Justification:Statement 1: It is the reverse. It will allow India to obtain natural gas.


So, it is wrong.
Statement 2: There has been no such announcement. Moreover, India may find it
difficult to extend the pipeline across China due to geographical barriers. Hence,
wrong.

H
TS

Learning:TAPI gas pipeline project is a proposed trans-country natural gas pipeline


from Caspian Sea, Turkmenistan to India through Afghanistan and Pakistan.
The estimated construction cost of the project is 10 billion US dollars and is being
funded by Asian Development Bank (ADB).

The TAPI Project is considered as an important initiative of these four countries to


connect energy rich Central Asia with energy starved South Asia. It marks a new
dawn of economic engagement through regional connectivity by economically
integrating region stretching from the Bay of Bengal to the Caspian Sea.

SI

Q Source:Q
Source:http://www.thehindu.com/news/national/tapi-is-a-dream-come-true-for-india
-energy-needs-vp-hamid-ansari/article7981061.ece

IN

13 What can the system of Proportional representation mean in the national context?
A. All constituencies in the country will have their political representative in the
Parliament.
B. All candidates fighting elections will be entitled to public office irrespective
of the number of votes they have garnered.
C. The seats in a constituency are distributed on the basis of votes polled by a
party, and not on individual basis.
D. Any system which is followed for elections to the offices of highest
constitutional functionaries
User Answer : C
Correct Answer : C
Answer Justification :

(C) Insights Active Learning. | All rights reserved.

www.insightsias.com

TEST - 7

User Name :

Sunanthaa

Total Marks

200

Mark Scored

84.67

Learning:There are several kinds of PR systems. In one of the systems, the entire
country is treated as one single constituency.
Voters vote for the party, not for the candidates.

IA
S

Seats are allocated to each party is proportion of its share of votes in the national
election.
Another variant of PR, the Single Transferable Vote system (STV), is followed for
Rajya Sabha elections. Every State has a specific quota of seats in the Rajya Sabha.
The members are elected by the respective State legislative assemblies. The voters
are the MLAs in that State.

H
TS

Q Source:Page 57:Chapter 3:11th NCERT: Indian Constitution

IN

SI

14 Consider the following about an important tributary of the Indus.


1. It rises from a spring at Verinag situated at the foot of the Pir Panjal in Kasmir
valley.
2. It flows through Srinagar and the Wular lake before entering Pakistan through a
deep narrow gorge
3. It joins the Chenab near Jhang in Pakistan.
The above refer to?
A. Satluj
B. Ravi
C. Beas
D. Jhelum
User Answer : D
Correct Answer : D
Answer Justification :
Learning:The Jhelum, an important tributary of the Indus, rises from a spring at
Verinag situated at the foot of the Pir Panjal in the south-eastern part of the valley of
Kashmir.
It flows through Srinagar and the Wular lake before entering Pakistan through a
deep narrow gorge. It joins the Chenab near Jhang in Pakistan.
Q Source:Page 25:Chapter 3:11th NCERT: India Physical Geography
15 Annual Pravasi Bharatiya Divas is celebrated on 9th January every year to mark the

(C) Insights Active Learning. | All rights reserved.

www.insightsias.com

10

TEST - 7

User Name :

Sunanthaa

Total Marks :

200

Mark Scored :

84.67

IA
S

contribution of Overseas Indian community in the development of India. January 9 was


chosen as the day to celebrate this occasion since
A. It was on this day that Mahatma Gandhi, a pravasi, returned to India from
South Africa
B. The First Pravasi moved from India on this date
C. This was the Date on which the first Non-resident citizen of India assumed a
public office
D. There is no particular reason behind the date.
User Answer :
Correct Answer : A
Answer Justification :

H
TS

Justification:January 9 was chosen as the day to celebrate this occasion since it was
on this day in 1915 that Mahatma Gandhi, the greatest Pravasi, returned to India
from South Africa, led Indias freedom struggle and changed the lives of Indians
forever.
Learning:The Pravasi Bhartiya Diwas is essentially a platform to connect this
Indian diaspora with the motherland. It was started in 2003.

The idea was to develop consciousness of the concept of the global Indian family,
define Indias new approach towards the Diaspora, provide a forum to discuss issues
and concerns of people of the Diaspora and to involve the Diaspora in the
development and advancement of India.

SI

During the event, individuals of exceptional merit are honoured with the prestigious
Pravasi Bharatiya Samman Award to appreciate their role in Indias growth. The
event also provides a forum for discussing key issues concerning the Indian
Diaspora.

IN

Q Source:http://moia.gov.in/services.aspx?id1=25&id=m1&idp=25&mainid=23

16 As industrial societies progress, there is considerable specialization of labour. Which of


the following is a pre-requisite for the specialization of labour?
A. Efficient and effective division of labour
B. Positive marginal product of labour
C. High turnover of labour
D. Labour Force Participation Rate reaches unity
User Answer : A
Correct Answer : A
Answer Justification :

(C) Insights Active Learning. | All rights reserved.

www.insightsias.com

11

TEST - 7

User Name :

Sunanthaa

Total Marks

200

Mark Scored

84.67

Justification:Positive marginal product of labour means each extra labour added to


the production process yields positive output. There may or may not be
specialization of labour here. So, (b) is wrong.
High turnover of labour is not related with the specialization of labour. It may comes
even via the use of machines, capital goods etc. So, (c) is wrong.

IA
S

Labour Force Participation Rate reaches unity means all eligible people search for
work or get work in the society. There may still be considerable diffusion of work
responsibilities, for e.g. in agriculture in rural India. So, (d) is wrong.

H
TS

Learning:Division of labour is the specialisation of work tasks, by means of which


different occupations are combined within a production system. All societies have at
least some rudimentary form of division of labour especially between the tasks
allocated to men and those performed by women.
With the development of industrialism, however, the division of labour became
more complex than in any prior type of production system. In the modern world, the
division of labour is international in scope
Q Source:Chapter 1: Page 19: 11th NCERT: Understanding Society

IN

SI

17 How does the Prime Minister of India act to build consensus in the Union Cabinet?
1. He can dismiss a minister who does not agree to the wish of the Cabinet.
2. He can override the decision of a minister to build consensus in Cabinet.
Which of the above is/are correct?
A. 1 only
B. 2 only
C. Both 1 and 2
D. None
User Answer : B
Correct Answer : C
Answer Justification :
Justification:In India, the Prime Minister enjoys a pre-eminent place in the
government. The Council of Ministers cannot exist without the Prime Minister. He
has the central responsibility of steering the cabinet and ensuring consensus.
If a cabinet minister disagrees with the cabinet frequently and even speaks out about
the differences in public, he may have to resign, or the PMY may dismiss him. So, 1
is correct.

(C) Insights Active Learning. | All rights reserved.

www.insightsias.com

12

TEST - 7

User Name :

Sunanthaa

Total Marks :

200

Mark Scored :

84.67

The Prime Minister is involved in all crucial decisions of the government and
decides on the policies of the government. Since he appoints the council of
ministers, he can override their decisions. So, 2 is correct.
Q Source:Page 90:Chapter 4:11th NCERT: Indian Constitution

H
TS

IA
S

18 A public bill is a bill which is


A. Introduced by any MP in national interest
B. Related to public finances
C. Related to the conduct of Business in the government
D. Introduced by a Minister
User Answer : A
Correct Answer : D
Answer Justification :
Q Source:Chapter 3:11th NCERT: Indian Constitution

IN

SI

19 How does the legislature control taxation and the way in which money is used by the
government?
1. All laws that require money to be withdrawn from the consolidated Fund of India
require the approval of the Parliament.
2. All laws that deal with financial matter must be authorized by the Public Accounts
Committee of the Parliament.
3. All laws that give the government additional taxation powers must be approved by
the Departments Standing Committees of the Parliament.
Select the correct answer using the codes below:
A. 1 and 3 only
B. 2 and 3 only
C. 1 only
D. 1, 2 and 3
User Answer : C
Correct Answer : C
Answer Justification :
Justification:Laws that deal with financial matters must be first examined by the
Speaker and then approved by the Parliament. PAC only examines government
spending via CAG reports. So, 2 is incorrect.
The Departments Standing Committees of the Parliament examine departmental
budgets after the annual budget is introduced in the Parliament. They do not have

(C) Insights Active Learning. | All rights reserved.

www.insightsias.com

13

TEST - 7

User Name :

Sunanthaa

Total Marks

200

Mark Scored

84.67

approval powers. So, 3 is wrong.


Q Source:Chapter 3:11th NCERT: Indian Constitution

IA
S

20 The Political neutrality of the Indian Bureaucracy is rooted in


A. The Articles establishing the Civil services in the Constitution
B. Several guidelines issued by the Department of Personnel and Training
C. A Memorandum of Understanding (MoU) signed between a Minister and
bureaucrat
D. None of the above
User Answer :
Correct Answer : D
Answer Justification :

H
TS

Justification:Political neutrality of a civil servant means, a) he will faithfully advise


the political executive without bias; b) he will carry out the instructions of the
representative without bias or his personal ideologies; and c) he shall keep himself
away from purely political matters.

This principle comes from bureaucratic conventions, code of conducts of civil


servants and to an extent the informal code of ethics. So, all options (a), (b), (c) are
wrong.
Q Source:Improvisation:Chapter 4:11th NCERT: Indian Constitution

IN

SI

21 A country is likely to have multiple time zones if


1. It is spread in a large geographical area longitude-wise
2. It is spread in a large geographical area latitude-wise.
3. It is closer to the equator.
4. If it is closer to the poles.
Which of the above is/are correct?
A. 1 only
B. 1 and 2 only
C. 1, 2 and 4 only
D. 2, 3 and 4 only
User Answer : A
Correct Answer : A
Answer Justification :
Justification: If a nation is spread on a large geographical area latitudinally, the Sun

(C) Insights Active Learning. | All rights reserved.

www.insightsias.com

14

TEST - 7

User Name :

Sunanthaa

Total Marks :

200

Mark Scored :

84.67

will be at different positions in the sky for different latitudes on the nation at the
same time. So, there is a possibility of having multiple time zones. For e.g. in
Russia, there are about 11 time zones. So, 1 is correct.
The same cannot be said for longitude as it does not concern with the movement of
the Sun or daylight. So, 2 is incorrect.

IA
S

By the same logic as given in statement 1, both 3 and 4 will be incorrect.


Q Source: Chapter 1: 11th NCERT: India Physical Geography

H
TS

22 A Gram Sabha is constituted by the


A. Gram Panchayat
B. State Election Commission
C. State Legislature
D. None of the above
User Answer : D
Correct Answer : D
Answer Justification :

SI

Learning: The Seventy-third Amendment of the Constitution of India, officially


known as The Constitution (Seventy-third Amendment) Act, 1992, added Part IX of
the Constitution of India, related to Panchayats, and the Eleventh Schedule to the
Constitution which deals with matters on which the Panchayats may be devolved
with powers and responsibility by the State Legislatures by law.

IN

Article 243A of the Constitution (Seventy-second) Amendment Bill, as introduced


in the House, proposed that a Gram Sabha may exercise such powers at the village
level as the Legislature of a State may, by law, provide. So, while GS is constituted
by the Constitution, only powers are given by the state legislature.
So, (d) will be correct, and (c) will be wrong.
Q Source: Chapter 8: 11th NCERT: Indian Constitution

23 Unconstitutional bills introduced by the Legislative Assembly of a State may be


disapproved by
1. The President of India after being referred to by the Governor of the State
2. The Rajya Sabha after being referred to by the Legislative Council of the State
Which of the above is/are correct?

(C) Insights Active Learning. | All rights reserved.

www.insightsias.com

15

TEST - 7

User Name :

Sunanthaa

Total Marks

200

Mark Scored

84.67

IA
S

A. 1 only
B. 2 only
C. Both 1 and 2
D. None
User Answer :
Correct Answer : A
Answer Justification :
Justification: The Governor usually reserves such bills for the President who then
either approves the bill or vetoes it or sends it back to the assembly. This is to ensure
harmony between the central laws, state laws and the constitution. So, 1 is correct.
The RS does not has such powers over state bills even though it represents the
states. So, 2 is incorrect.

H
TS

Q Source: Improvisation: Chapter 5: 11th NCERT: Indian Constitution

IN

SI

24 Which of the following fall under the Peninsular block of India?


1. Karbi Anglong Plateau
2. Rajmahal Hills
3. Meghalaya Plateau
Select the correct answer using the codes below:
A. 1 and 2 only
B. 2 and 3 only
C. 1 only
D. 1, 2 and 3
User Answer : A
Correct Answer : D
Answer Justification :
Learning: The northern boundary of the Peninsular Block may be taken as an
irregular line running from Kachchh along the western flank of the Aravali Range
near Delhi and then roughly parallel to the Yamuna and the Ganga as far as the
Rajmahal Hills and the Ganga delta.
Apart from these, the Karbi Anglong and the Meghalaya Plateau in the northeast and
Rajasthan in the west are also extensions of this block. The northeastern parts are
separated by the Malda fault in West Bengal from the Chotanagpur plateau. In
Rajasthan, the desert and other desertlike features overlay this block.
Q Source: Page 8: Chapter 2: 11th NCERT: India Physical Geography

(C) Insights Active Learning. | All rights reserved.

www.insightsias.com

16

TEST - 7

User Name :

Sunanthaa

Total Marks :

200

Mark Scored :

84.67

IA
S

25 Consider the following statements.


1. It is known for its fast-flowing rivers such as Tista.
2. It is a region of high mountain peaks like Kanchenjunga.
3. The higher reaches of this region are inhabited by Lepcha tribes.
The above refer to which region?
A. The Darjiling and Sikkim Himalayas
B. Pir Panjal
C. Nubra valley
D. Uttaranchal Himalayas
User Answer : A
Correct Answer : A
Answer Justification :

H
TS

Justification: Statement 1 and 2: It is a region of high mountain peaks like


Kanchenjunga (Kanchengiri), and deep valleys and thus also known for its fastflowing rivers such as Tista.
Statement 3: The higher reaches of this region are inhabited by Lepcha tribes while
the southern part, particularly the Darjiling Himalayas, has a mixed population of
Nepalis, Bengalis and tribals from Central India

Q Source: Page 13: Chapter 2: 11th NCERT: India Physical Geography

IN

SI

26 The writ of Quo Warranto can be issued to address which of the following?
A. If someone has been detailed illegally
B. If a legislator holds such office which makes him ineligible to become an MP
C. If an official is not doing her duties properly
D. If a lower court has usurped illegal jurisdiction
User Answer : B
Correct Answer : B
Answer Justification :
Learning:Quo warranto means by what warrant? This means that Supreme Court
and High Court may issue the writ which restrains the person or authority to act in
an office which he / she is not entitled to.
This writ is applicable to the public offices only.
Q Source: Page 131: Chapter 6: 11th NCERT: Indian Constitution

(C) Insights Active Learning. | All rights reserved.

www.insightsias.com

17

TEST - 7

User Name :

Sunanthaa

Total Marks

200

Mark Scored

84.67

IA
S

27 Which of the following committees of Parliament scrutinize the activities of the


executive?
1. Committee on Subordinate Legislation
2. Committee on Government Assurances
3. Committee on Papers Laid on the Table
Select the correct answer using the codes below:
A. 1 and 3 only
B. 2 and 3 only
C. 3 only
D. 1, 2 and 3
User Answer :
Correct Answer : D
Answer Justification :

H
TS

Justification: Statement 1: This committee examines and reports to the House


whether the powers to make regulations, rules, sub-rules and bye-laws delegated by
the Parliament or conferred by the Constitution to the Executive are being properly
exercised by it.
Statement 2: This committee examines the assurances, promises and undertakings
given by ministers from time to time on the floor of the House and reports on the
extent to which they have been carried through.

SI

Statement 3: It examines all papers laid on the table of the House by ministers to see
whether they comply with provisions of the Constitution, or the related Act or Rule.
It does not examine statutory notifications and orders that fall under the jurisdiction
of the Committee on Subordinate Legislation.

IN

Q Source: Improvisation: Chapter 5: 11th NCERT: Indian Constitution

28 Peninsular rivers are characterised by absence of meanders because


1. These rivers do not have capacity for lateral erosion of banks.
2. There rivers pass through hard rocky terrain.
Which of the above is/are correct?
A. 1 only
B. 2 only
C. Both 1 and 2
D. None
User Answer : C
Correct Answer : B

(C) Insights Active Learning. | All rights reserved.

www.insightsias.com

18

TEST - 7

User Name :

Sunanthaa

Total Marks :

200

Mark Scored :

84.67

Answer Justification :
Justification: Any rivers that flow at a considerable speed can erode both lateral
banks as well as river bed. So, 1 is wrong as peninsular rivers flow at fast speeds.
When they slow down, lateral bank cutting is predominant.

IA
S

However, this erosion can take place only when the bank is soft or river bed is made
of erodible soft rocks. Since, peninsular blocks are hard, rocky, old and
consolidated, little silt is accumulated by these rivers. So, 2 is correct.
Q Source: Page 27: Chapter 3: 11th NCERT: India Physical Geography

H
TS

29 The Manishankar Aiyer committee appointed by the government sometime back was
related with
A. Reforms in the Civil Services Institutions
B. Reforms in the Local bodies
C. Reforms in the Governance of Public Sector Undertakings (PSUs)
D. An overhaul of the regulatory architecture of the Union government
User Answer :
Correct Answer : B
Answer Justification :

Learning: The Manishankar Aiyer committee was setup to show the way forward.
It recommended the following:

IN

SI

Panchayats should be allowed to raise their own resources through collection


of taxes.
Panchayat heads and members should get training in accounts and audit,
besides governance.
Panchayats should have effective control over their employees( deputed to
them by the state) though payment control
A single ministry should be constituted to look after both Panchayats and
municipalities
An ombudsman should be setup for Panchayats for all states.

As per the committee, complete and effective operationalizing of the panchayati raj
system has been hobbled mainly by a lack of clarity as to how to proceed
scientifically, objectively, methodically and practically down the path of
devolution.
Q Source: : Due to the panchayat issues, it is sometimes in news.

(C) Insights Active Learning. | All rights reserved.

www.insightsias.com

19

TEST - 7

User Name :

Sunanthaa

Total Marks

200

Mark Scored

84.67

http://www.newindianexpress.com/nation/Panchayati-Raj-Institutions-given-a-raw-d
eal-Mani-Shankar-Aiyar/2013/04/25/article1560172.ece

H
TS

IA
S

30 Which of the following is/are related to deliberation/discussion in the Parliament?


1. Adjournment motion
2. Zero Hour
3. Answer Hour
Select the correct answer using the codes below:
A. 2 only
B. 1 and 3 only
C. 1 and 2 only
D. 1, 2 and 3 only
User Answer : A
Correct Answer : C
Answer Justification :

Justification: Statement 1: A motion for an adjournment of the business of the


House for the purpose of discussing a definite matter of urgent public importance
may be made with the consent of the Speaker.

Statement 2: Zero Hour in Parliament starts at 12 noon during which members raise
matters of importance, especially those that cannot be delayed. Nobody knows
which issue a member would raise during this hour. As a result, questions so raised
without prior notice may be of no importance.

SI

Q Source: Chapter 5: 11th NCERT: Indian Constitution

IN

31 In the Cold Weather Season in India, which of the following can be the reasons for the
excessive cold in North India?
1. Continentality
2. Snowfall in the nearby Himalayan ranges
3. Cold winds coming from the Central Asian region
Select the correct answer using the codes below:
A. 1 and 2 only
B. 1 and 3 only
C. 2 only
D. 1, 2 and 3
User Answer : D
Correct Answer : D
Answer Justification :

(C) Insights Active Learning. | All rights reserved.

www.insightsias.com

20

TEST - 7

User Name :

Sunanthaa

Total Marks :

200

Mark Scored :

84.67

Justification: Continentality implies distance from sea and landmass being


surrounded by only land. Due to the absence of moderation effect of sea on climate,
extreme temperatures are found. So, 1 is correct.
Snowfall in the nearby Himalayan ranges and cold winds from CA region reduce the
temperature in the northern regions, so, 2 is correct.

IA
S

Q Source: Page 40: Chapter 4: 11th NCERT: India Physical Geography

H
TS

32 The Representation of Peoples Act 1951 (RPA) deals with


1. Qualification of voters
2. Delimitation of constituencies
3. Administrative machinery for conducting elections
Select the correct answer using the codes below:
A. 1 and 3 only
B. 2 only
C. 1, 2 and 3
D. 3 only
User Answer :
Correct Answer : D
Answer Justification :

Justification & Learning: Representation of Peoples Act 1950 (RPA Act 1950)
provides for the following:

IN

SI

Qualification of voters.
Preparation of electoral rolls.
Delimitation of constituencies.
Allocation of seats in the Parliament and state legislatures.

RPA Act 1951 provides for


Actual conduct of elections.
Administrative machinery for conducting elections.
Poll
Election offences.
Election disputes.
By-elections.
Registration of political parties.

Q Source: Improvisation: Chapter 3: 11th NCERT: Indian Constitution

(C) Insights Active Learning. | All rights reserved.

www.insightsias.com

21

TEST - 7

User Name :

Sunanthaa

Total Marks

200

Mark Scored

84.67

IA
S

33 The Eleventh degree channel divides


A. Lakshadweep and Minicoy
B. The Andaman in the north and the Nicobar in the south
C. Daman ports from Diu ports
D. None of the above
User Answer : C
Correct Answer : A
Answer Justification :

Learning: The islands of the Arabian sea include Lakshadweep and Minicoy. These
are scattered between 8N-12N and 71E -74E longitude. These islands are
located at a distance of 280 km-480 km off the Kerala coast. The entire island group
is built of coral deposits.

H
TS

There are approximately 36 islands of which 11 are inhabited. Minicoy is the largest
island with an area of 453 sq. km. The entire group of islands is broadly divided by
the Eleventh degree channel, north of which is the Amini Island and to the south of
the Canannore Island. The Islands of this archipelago have storm beaches consisting
of unconsolidated pebbles, shingles, cobbles and boulders on the eastern seaboard.
Q Source: Page 19: Chapter 2: 11th NCERT: India Physical Geography

IN

SI

34 While entering the plains, Himalayan Rivers generally make which of the following
patterns?
1. Braided channels
2. Flat valleys
3. Indented Coastlines
4. Ox-bow lakes
Select the correct answer using the codes below:
A. 1 and 2 only
B. 2 and 4 only
C. 1 and 3 only
D. 1, 2 and 4 only
User Answer : D
Correct Answer : D
Answer Justification :
Justification: Statement 1: Braided channels occur in rivers with high slope when,
threshold level of sediment load or slope is reached, any slope over this threshold
creates a braided stream. This is true for Himalayan Rivers and so 1 is correct.

(C) Insights Active Learning. | All rights reserved.

www.insightsias.com

22

TEST - 7

User Name :

Sunanthaa

Total Marks :

200

Mark Scored :

84.67

Statement 2: Flat valleys, like V-shaped valleys, are formed by streams, but they are
no longer in their youthful stage, and are instead considered mature. With these
streams, as the slope of a stream's channel becomes smooth, and begins to exit the
steep V or U-shaped valley, the valley floor gets wider. Because the stream gradient
is moderate or low, the river begins to erode the bank of its channel instead of valley
walls. So, 2 is correct as it is found in Himalayan rivers.

IA
S

Statement 4: An oxbow is a crescent-shaped lake lying alongside a winding river.


The oxbow lake is created over time as erosion and deposits of soil change the
river's course. This is found in bank eroding Himalayan Rivers. So, 4 is correct.

Q Source: Page 25: Chapter 3: 11th NCERT: India Physical Geography

H
TS

35 Which of the following bodies was entirely nominated by the British?


A. Constituent Assembly, 1946
B. Drafting Committee of the Constitution
C. First Interim Cabinet of India after formation of constitution
D. None of the above
User Answer : C
Correct Answer : D
Answer Justification :

SI

Justification: Constituent Assembly, 1946 was partly nominated, partly elected. So


you can easily eliminate. Option (a) is wrong.
Drafting Committee of the Constitution was appointed by the assembly and the
leaders. So, (b) is wrong.

IN

First Interim Cabinet of India after formation of constitution was formed based on
election results and party politics. So, (c) is wrong.
Thus, (d) will be the only answer.
Q Source: Chapter 9: 11th NCERT: Indian Constitution

36 The southwest monsoon first arrives India at the


A. Odisha Coast
B. TamilNadu and Andhra Pradesh Coasts
C. Kerala Coast

(C) Insights Active Learning. | All rights reserved.

www.insightsias.com

23

TEST - 7

User Name :

Sunanthaa

Total Marks

200

Mark Scored

84.67

D. Maharashtra Coast
User Answer : C
Correct Answer : C
Answer Justification :

IA
S

Learning: As a result of rapid increase of temperature in May over the northwestern plains, the low pressure conditions over there get further intensified. By
early June, they are powerful enough to attract the trade winds of Southern
Hemisphere coming from the Indian Ocean.
These southeast trade winds cross the equator and enter the Bay of Bengal and the
Arabian Sea, only to be caught up in the air circulation over India. Passing over the
equatorial warm currents, they bring with them moisture in abundance.

H
TS

After crossing the equator, they follow a south-westerly direction. That is why they
are known as southwest monsoons.
Q Source: Page 45: Chapter 4: 11th NCERT: India Physical Geography

IN

SI

37 Consider the following about the office of Chief Secretary (CS) in the State
Government.
1. It is not mentioned in the Constitution.
2. The CS acts as the secretary to the State cabinet.
Which of the above is/are correct?
A. 1 only
B. 2 only
C. Both 1 and 2
D. None
User Answer : C
Correct Answer : C
Answer Justification :
Learning: About Chief Secy:
Administrative head of state admin In India.
In many ways counterpart of Cabinet Secy.
Chief Coordinator of State secretariat and other administrative departments
Attends cabinet meetings
Role in policy formulation both formal and informal substantial + supervisory
role
Head of Civil Services in the state and Cabinet Secretariat

(C) Insights Active Learning. | All rights reserved.

www.insightsias.com

24

TEST - 7

User Name :

Sunanthaa

Total Marks :

200

Mark Scored :

84.67

Status equal to that of Secretary to the GoI


Different from Principal Secy to the CM (he may also attend cabinet
meetings)
Functions

H
TS

IA
S

Not mentioned in the Constitution, but in Rules of Business of State govt.


Some functions customs and convention
Alter-ego of the CM
Secy. to the cabinet
Head of Cabinet Secretariart
Role in Policy formulation - ensuring implementation of decisions
(supervisory role - whereas Cabinet Secy only coordination)
Matters affecting centre-state and inter-state relations (like water disputes);
zonal councils (secy.)
All important decisions regarding adoption of new schemes
Head of a few administrative depts. also like DoPT, Planning dept.
(traditionally put under him - also 1st ARC) (cabinet secy only head of
cabinet secretariat)
Responsible for personnel matters of state cadre officers of the IAS, and any
other services if given +Admin. Reforms
Consulted by CM in appointing members of SPSCs, Lokayuktas in some
states.

SI

Q Source: Often in news, and based on previous year questions on Cabinet


Secretary

IN

38 The District Planning Committee is a


A. Constitutional authority expected to consolidate development plans made by
Rural and urban local governments.
B. Statutory authority expected to assist the State Planning Departments in
assessing the resources of particular districts
C. An executive authority expected to help the panchayats in the implementation
of social welfare schemes
D. None of the above
User Answer : B
Correct Answer : A
Answer Justification :
Justification:

(C) Insights Active Learning. | All rights reserved.

www.insightsias.com

25

TEST - 7

User Name :

Sunanthaa

Total Marks

200

Mark Scored

84.67

Learning: Issues with DPCs:

IA
S

It is a constitutional authority expected to frame development plans under


Article 243ZD and consolidate plans made by Rural and urban local
governments.
4/5th of total members at least to be elected by, and from amongst, the
members of the panchayats + right proportion of municipality and villages +
rest 1/5th experts, specialists, govt. representatives can be included
State legislatures determine - chairperson of DPC, other parameters

SI

H
TS

Only half the states have functioning DPCs


State planning board guidelines not followed e.g. in Rajasthan
Do not have powers to modify or change existing plans or budget or projects
- can only offer suggestions - ultimate decision local body
Most programmes implemented in isolation from DPCs (without seeking
advice) like by DRDA, panchayats etc.
Can not match the technical sophistication or planners, engineers and
economists like central or state planning boards - due to lack of skilled
personnel at district level
State FCs have rarely recommended funds for district plans + no provision in
state budgets also
Most district plans are a mere summation of ULBs and RLBs plans and not a
synergistic integration.
Tied funds given by centre - now flexi-funds given only to the State Planning
departments
MPLAD and MLA spending is not integrated with the district plans.
Q Source: Chapter 8: 11th NCERT: Indian Constitution

IN

39 Chabahar port is located in Sistan-Balochistan Province on Iran's southeastern coast and


is of great strategic utility for India. It will
1. Allow sea-land access route to Afghanistan bypassing Pakistan.
2. Allow India to control and regulate the shipping trade of South Asia en route.
Which of the above is/are correct?
A. 1 only
B. 2 only
C. Both 1 and 2
D. None
User Answer : B
Correct Answer : A

(C) Insights Active Learning. | All rights reserved.

www.insightsias.com

26

TEST - 7

User Name :

Sunanthaa

Total Marks :

200

Mark Scored :

84.67

Answer Justification :

H
TS

IA
S

Justification: The diagram below (taken from the Q link) shows it all.

SI

Statement 2 will be incorrect as Chabahar port is not a choke point (like Strait of
Hormuz). You can see the location in the map above.

IN

Q Source:
http://www.thehindu.com/news/national/decks-cleared-for-indias-role-in-irans-chab
ahar-port/article7583968.ece

40 Which of the following institutions hold a controlling stake in the International


Monetary Fund (IMF)?
1. Asian Development Bank (ADB)
2. Asia Pacific Economic Council (APEC)
3. BRICS
Select the correct answer using the codes below:
A. 1 and 3 only
B. 2 and 3 only
C. 1 only

(C) Insights Active Learning. | All rights reserved.

www.insightsias.com

27

TEST - 7

User Name :

Sunanthaa

Total Marks

200

Mark Scored

84.67

D. None of the above


User Answer :
Correct Answer : D
Answer Justification :

ADB was established in 1997. So, 1 is incorrect.


APEC in 1989, so 2 is incorrect.
BRICS in 2005 only, so 3 is incorrect.

IA
S

Justification: All the three organizations mentioned above did not exist when IMF
was formed at the Bretton Woods Conference in 1945.

H
TS

IMF consists of 188 member nations with varying voting rights. The voting quota
issue reforms remains at the core of IMF reforms.
Q Source:
http://www.newindianexpress.com/business/news/IMF-has-Important-Role-in-Glob
al-Economy-Jaitley/2015/10/10/article3072772.ece

IN

SI

41 which of the following correctly differentiate(s) between Non-Resident Indian (NRI)


and an Overseas Citizen of India (OCI)?
1. OCI is a kind of dual citizenship in India, but NRI is not.
2. NRIs enjoy the right to vote, but OCIs do not.
3. OCIs have to obtain a visa every time they visit India, which is not the case with
NRIs.
Select the correct answer using the codes below:
A. 1 and 3 only
B. 2 and 3 only
C. 2 only
D. None of the above
User Answer :
Correct Answer : C
Answer Justification :
Justification & Learning:This will clarify the differences between NRIs vs PIOs vs
OCI. Recently PIO and OCI categories have been nearly merged.
ANon-Resident Indian(NRI) is acitizen of Indiawho holds an
Indianpassportand has temporarilyemigratedto another country for six

(C) Insights Active Learning. | All rights reserved.

www.insightsias.com

28

TEST - 7

User Name :

Sunanthaa

Total Marks :

200

Mark Scored :

84.67

months
or more for employment, residence, education or any other purpose.

H
TS

IA
S

APerson of Indian Origin(PIO) is a person of Indian origin or ancestry but


who is not a citizen of India and is the citizen of another country (except
South Asian countries). A PIO might have been a citizen of India and
subsequently taken the citizenship of another country, or have ancestors born
in India or other states.
A Overseas Citizen of India (OCI) is a PIO who had migrated to other
countries and acquired citizenship (except Afghanistan and Bangladesh)
there. If their country of residence allows dual citizenship they are eligible to
hold a OCI card.
Overseas Citizenship of India is not an actual citizenship of India and
thus, does not amount to dual citizenship or dual nationality.
An Overseas Citizen of India will enjoy all rights and privileges
available toNon-Resident Indianson a parity basis excluding the
right to invest in agriculture and plantation properties or hold public
office, and the right to vote.
Q Sources: It had been a hot topic for sometime last year and this year too and
needed to be covered.

http://www.thehindu.com/news/modi-announces-lifelong-visas-for-indian-diaspora/
article6456543.ece

SI

http://www.thehindu.com/news/cities/bangalore/oci-students-can-write-jee-seek-entr
y-to-iits-under-general-quota/article7592529.ece

IN

42 Most of the large Peninsular Rivers have their origin in


A. Eastern Ghats
B. Western Ghats
C. Vindhyan ranges
D. Delhi Ridge
User Answer : B
Correct Answer : B
Answer Justification :
Justification:Large Rivers flowing on the peninsular plateau have their origin in the
Western Ghats and discharge their waters in the Bay of Bengal.

(C) Insights Active Learning. | All rights reserved.

www.insightsias.com

29

TEST - 7

User Name :

Sunanthaa

Total Marks

200

Mark Scored

84.67

The Narmada and Tapi are two large rivers which are exceptions. They along with
many small rivers discharge their waters in the Arabian Sea.
The Narmada originates on the western flank of the Amarkantak plateau.
Tapi originates from Multai in the Betul district of Madhya Pradesh.

IA
S

Q Source:Page 27: Chapter 3: 11th NCERT: India Physical geography

H
TS

43 Consider the following statements.


1. Its vegetation includes dry scrubs, dry and moist deciduous, wet evergreen forests,
sholas, grasslands and swamps.
2. It includes the largest known population of two endangered animal species, namely
the Nilgiri Tahr and the Lion-tailed macaque.
3. A majority of the flowering plants reported from the Western Ghats occur here.
The above refer to?
A. Periyar National Park
B. Eravikulam National Park
C. Nilgiri Biosphere Reserve
D. Palani Hills National Park and Wildlife Sanctuary
User Answer : C
Correct Answer : C
Answer Justification :

SI

Learning:The Nilgiri Biosphere Reserve (NBR), the first biosphere reserve of


India, was established in 1986. It embraces the sanctuary complex of Wyanad,
Nagarhole, Bandipur and Mudumalai, the entire forested hill slopes of Nilambur, the
Upper Nilgiri plateau, Silent Valley and the Siruvani hills.

IN

The largest south Indian population of elephant, tiger, gaur, sambar and chital as
well as a good number of endemic and endangered plants are also found in this
reserve.
Q Source: Page 66: Chapter 5: 11th NCERT: India Physical geography

44 Which of the following is/are initiatives of the Indian government concerned with
Indian Diaspora?
1. Know INDIA program
2. Indian Community Welfare Fund
3. Pravasi Yugantar Yojana

(C) Insights Active Learning. | All rights reserved.

www.insightsias.com

30

TEST - 7

User Name :

Sunanthaa

Total Marks :

200

Mark Scored :

84.67

IA
S

Select the correct answer using the codes below:


A. 1 and 2 only
B. 2 and 3 only
C. 1 and 3 only
D. All of the above
User Answer : D
Correct Answer : A
Answer Justification :

Justification:Statement 1: Know India Programme of the Ministry is a three-week


orientation programme for diaspora youth conducted with a view to promote
awareness on different facets of life in India and the progress made by the country in
various fields e.g. economic, industrial, education, Science & Technology.

H
TS

Statement 2: The Indian Community Welfare Fund (ICWF) is aimed at providing


'on site' welfare services on a means tested basis in the most deserving cases
including:

Boarding and lodging for distressed overseas Indian workers in Household /


domestic sectors and unskilled labourers;
Extending emergency medical care to the overseas Indians in need;
Providing air passage to stranded overseas Indians in need;
Providing initial legal assistance to the overseas Indians in deserving cases
etc.

SI

Statement 3:There is no such Yojana.


Q Source: http://moia.gov.in/services.aspx?id1=345&idp=345&mainid=73

IN

http://moia.gov.in/services.aspx?ID1=42&id=m4&idp=42&mainid=23

45 Which of the following describes the selection of the Vice-President of India most
appropriately?
A. Based on the discretion of the President of India
B. Based on the recommendation of the Union Cabinet agreed to by the Prime
Minister of India
C. Based on the votes garnered by the Leader of Oppositions of both Lok Sabha
and Rajya Sabha
D. Based on voting by a collegium of the Parliament
User Answer : B
Correct Answer : D

(C) Insights Active Learning. | All rights reserved.

www.insightsias.com

31

TEST - 7

User Name :

Sunanthaa

Total Marks

200

Mark Scored

84.67

Answer Justification :
Learning:The Vice President is elected for five years. His election method is
similar to that of the President, the only difference is that members of State
legislatures are not part of the electoral college.

IA
S

The Vice President may be removed from his office by a resolution of the Rajya
Sabha passed by a majority and agreed to by the Lok Sabha. The Vice President acts
as the ex-officio Chairman of the Rajya Sabha and takes over the office of the
President when there is a vacancy by reasons of death, resignation, removal by
impeachment or otherwise.
The Vice President acts as the President only until a new President is elected.

H
TS

Q Source: Page 88: Chapter 4: 11th NCERT: Indian Constitution

IN

SI

46 Which of the following parts of the constitution propagate/reflect certain social and
moral values?
1. Fundamental Duties
2. Preamble
3. Directive Principles of State Policy
4. Fundamental Rights
Select the correct answer using the codes below:
A. 1 and 2 only
B. 2, 3 and 4 only
C. 1 and 3 only
D. 1, 2, 3 and 4
User Answer : D
Correct Answer : D
Answer Justification :
Justification:FDs contain duties like fraternity, respecting women, national flag,
freedom struggle values, conserving environment. So, 1 is correct.
Preamble is the essence of the values of our constitution. So, 2 is correct.
DPSP contains a set of welfare ideals for the Indian state. So, 3 is correct.
FRs contain certain values like value of human life and dignity, values against
exploitation, values of equality, secularism etc. So, 4 is correct.

(C) Insights Active Learning. | All rights reserved.

www.insightsias.com

32

TEST - 7

User Name :

Sunanthaa

Total Marks :

200

Mark Scored :

84.67

Q Source:Chapter 2: 11th NCERT: Indian Constitution

H
TS

IA
S

47 India attaches a lot of importance to its Act East Policy given the region's international
and domestic significance. However, Look West Policy is also significant for us because
1. Gulf Cooperation Council (GCC) is our largest trading partner.
2. Our main air corridors and sea lanes for our connectivity with Europe and North
America lie through this area.
Which of the above is/are correct?
A. 1 only
B. 2 only
C. Both 1 and 2
D. None
User Answer : C
Correct Answer : C
Answer Justification :
Justification:Statement 1: As a single trading partner, GCC is our largest partner,
with almost $160 billion trade. If we take into account our trade with two other
countries in the Gulf region, Iraq ($21.5 billion) and Iran ($15 billion) the trade with
the region becomes $195 billion.

Additionally we get $30 billion annually as remittances from the GCC countries.
Add all this up and our economic relationship with the region becomes $225 billion.
This is more than the combined trade with ASEAN and China; or entire EU and US.

IN

SI

Statement 2: Straits of Malacca may be important for our trade with China, Japan,
South Korea and Taiwan but about 50% of our trade has to use the Straits of
Hormuz, Bab el Mandeb the narrow entrance to the red Sea, Suez Canal and the
Straits of Gibraltar, all located in this region. Air flights to Europe and North
America must use air corridors through this area for cost effectiveness.
Q Source:http://www.mea.gov.in/distinguished-lectures-detail.htm?154
http://www.thehindu.com/opinion/lead/sanjaya-baru-writes-the-sprouting-of-the-loo
k-west-policy/article7554403.ece

48 South-west Monsoon upto/in India is/are guided by


1. Trade Winds
2. Western Disturbances
3. Low pressure trough created over India

(C) Insights Active Learning. | All rights reserved.

www.insightsias.com

33

Select the correct answer using the codes below:


A. 1 and 2 only
B. 2 and 3 only
C. 1 only
D. 1 and 3
User Answer : D
Correct Answer : D
Answer Justification :

Sunanthaa

Total Marks

200

Mark Scored

84.67

IA
S

TEST - 7

User Name :

Justification:As a result of rapid increase of temperature in May over the northwestern plains, the low pressure conditions over there get further intensified. By
early June, they are powerful enough to attract the trade winds of Southern
Hemisphere coming from the Indian Ocean. So, 1 and 3 are correct.

H
TS

These southeast trade winds cross the equator and enter the Bay of Bengal and the
Arabian Sea, only to be caught up in the air circulation over India. Passing over the
equatorial warm currents, they bring with them moisture in abundance.
Western Disturbances withdraw when Monsoons come in India. However, the
tropical cyclonic depressions formed in

Bay of Bengal affect the direction and intensity of Monsoons in India. So, 3 is
incorrect.

SI

Q Source:Page 45: Chapter 4: 11th NCERT: India Physical Geography

IN

49 Why is having a written constitution important for a polity like India?


1. A democratic polity cannot be run without a written constitution.
2. A written constitution reduces the chances of disputes between the different organs
of the polity.
3. A written constitution is essential to a federal polity.
Select the correct answer using the codes below:
A. 1 and 2 only
B. 2 and 3 only
C. 1 and 3 only
D. 1, 2 and 3
User Answer : B
Correct Answer : B
Answer Justification :
Justification:Statement 1: It can be. Britain does not have a written constitution.

(C) Insights Active Learning. | All rights reserved.

www.insightsias.com

34

TEST - 7

User Name :

Sunanthaa

Total Marks :

200

Mark Scored :

84.67

Statement 2 & 3: A written constitution is actually more important when there are
several tiers of government, and where all tiers are supposed to be semi-autonomous
from each other. It reduces the chance of conflicts between them and ensures a
smooth functioning of federal polity.

IA
S

Q Source:Chapter 1 and 7: 11th NCERT: Indian Constitution

H
TS

50 Winter monsoons do not generally cause rainfall in India as


1. The monsoon winds move from land to sea.
2. They do not have the support of Western cyclonic disturbances.
Which of the above is/are correct?
A. 1 only
B. 2 only
C. Both 1 and 2
D. None
User Answer : C
Correct Answer : A
Answer Justification :

Justification:Winter monsoons do not cause rainfall as they move from land to the
sea. It is because firstly, they have little humidity; and secondly, due to anti cyclonic
circulation on land, the possibility of rainfall from them reduces.

SI

Western cyclonic disturbances are 'cyclonic' and not 'anti-cyclonic'. So, 1 is correct,
but 2 is incorrect. As you will see below, Western disturbances actually help to
bring rainfall.
So, most parts of India do not have rainfall in the winter season.

IN

Learning: However, there are some exceptions to it:


In north-western India, some weak temperate cyclones from the
Mediterranean Sea cause rainfall in Punjab, Haryana, Delhi and western Uttar
Pradesh.
Central parts of India and northern parts of southern Peninsula also get winter
rainfall occasionally.
Arunachal Pradesh and Assam in the northeastern parts of India also have
rains between 25 mm and 50 mm during these winter months.
During October and November, northeast monsoon while crossing over the
Bay of Bengal, picks up moisture and causes torrential rainfall over the Tamil
Nadu coast, southern Andhra Pradesh, southeast Karnataka and southeast

(C) Insights Active Learning. | All rights reserved.

www.insightsias.com

35

TEST - 7

User Name :

Sunanthaa

Total Marks

200

Mark Scored

84.67

Kerala.
Q Source: Page 45: Chapter 4: 11th NCERT: India Physical Geography

H
TS

IA
S

51 The Sustainable Development Goals (SDGs) adopted from 2015 do NOT deal with?
1. Education for all
2. Reduced inequalities among citizens
3. Peace, Justice and Strong Institutions
Select the correct answer using the codes below:
A. 1 only
B. 2 and 3 only
C. 3 only
D. They deal with all of the above
User Answer : D
Correct Answer : D
Answer Justification :
Justification: SDGs consist of 17 goals and 169 targets in contrast to 8 goals and 18
targets in MDGs set to expire in 2015.

It will apply to all UN member states and not just those in the developing world. In
this way, we hope they will become as "universal" as the Universal Declaration of
Human Rights-a vital element of the civic armoury in the fight for fairness.

SI

Goals are related to ending poverty; sustainable development; universal enrolment;


improving implementation etc. These goals are similar to MDGs, while others like
reducing inequality are new to SDGs.

IN

Learning: What is required for SDGs success?


MDGs success, in part, lie in China reducing poverty by nearly 30% since the
beginning of MDG in 2000. Similarly, experts opine, India's performance
will be crucial to the success of SDGs.
As per WB President, one state, UP, in India houses 8% of world's BPL
population. UP's progress will push SDGs success.
Instead of top-down approach in implementation of SDGs, bottom-up
approach is needed. Hence, state and local governments in participation with
civil society (NGOs, think tanks, social activists) will achieve greater
progress than Central government alone can do.
NITI Aayog, India, can play a crucial role in monitoring SDGs progress.
Ministries and departments can align social welfare scheme with SDGs. This

(C) Insights Active Learning. | All rights reserved.

www.insightsias.com

36

TEST - 7

User Name :

Sunanthaa

Total Marks :

200

Mark Scored :

84.67

will lessen the administrative burden of implementation of SDGs.


Q Source:
http://www.un.org/sustainabledevelopment/sustainable-development-goals/

H
TS

IA
S

52 The power of Judicial Review and independence of Judiciary in the Indian constitution
had been
A. An indigenous idea given by the Political affairs committee of the
constitution
B. For long in the mind of Nationalists and public alike
C. Generated in the course of the Civil Disobedience Movement, 1930s
D. Borrowed from the USA constitution and improvised
User Answer :
Correct Answer : D
Answer Justification :
Justification: It comes from the American constitution. However, we have not
adopted the 'due process of law' and went for the 'procedure established by law' in
handling Judicial review.

The former means that the Judiciary can review laws even based on reasonableness.
In the latter, the Judiciary only checks whether due process was followed in enacting
a law.

SI

However, in India, the lines between the two have blurred over the years especially
since the Kesavananda Bharati case 1973. Therefore, we see both in play today,
which is often called as Judicial activism.

IN

Q Source:Chapters 9 and 10: 11th NCERT: Indian Constitution

53 The rivers originating from the Amarkantak range present a good example of
A. Dendritic Drainage pattern
B. Radial Drainage pattern
C. Trellis Drainage pattern
D. Centripetal Drainage pattern
User Answer : B
Correct Answer : B
Answer Justification :

(C) Insights Active Learning. | All rights reserved.

www.insightsias.com

37

TEST - 7

User Name :

Sunanthaa

Total Marks

200

Mark Scored

84.67

Justification: When the rivers originate from a hill and flow in all directions, the
drainage pattern is known as 'radial'. This can be seen in the Amarkantak range. So,
(b) is correct.
Learning: The drainage pattern resembling the branches of a tree is known as
"dendritic" the examples of which are the rivers of northern plain.

IA
S

When the primary tributaries of rivers flow parallel to each other and secondary
tributaries join them at right angles, the pattern is known as 'trellis'.

(When the rivers discharge their waters from all directions in a lake or depression,
the pattern is known as 'centripetal'.

H
TS

Q Source: Chapter 3: 11th NCERT: India Physical Geography

SI

54 Why are the western coastal plains devoid of any delta?


1. Coasts are shallow.
2. The plains are indented.
3. They are a region of high tectonic activity.
Select the correct answer using the codes below:
A. 1 only
B. 2 and 3 only
C. 1 and 3 only
D. None of the above
User Answer : A
Correct Answer : D
Answer Justification :

IN

Justification: The Rivers that originate in Western Ghats flow eastwards or


westwards.
The west flowing rivers are shorter than the east flowing rivers. There is a high
gradient for the west flowing rivers than the east flowing rivers.
More gradient means higher is the velocity of the river. Therefore West flowing
rivers end up forming estuaries rather than deltas.
Moreover, the rocks are hard to erode which means less silt and little delta formation
at sea mouth.
None of the reasons 1, 2 or 3 addresses these reasons. Even when coasts are shallow

(C) Insights Active Learning. | All rights reserved.

www.insightsias.com

38

TEST - 7

User Name :

Sunanthaa

Total Marks :

200

Mark Scored :

84.67

or indented or of high tectonic activity, deltas can be formed.


Q Source:Chapter 3: 11th NCERT: India Physical Geography

H
TS

IA
S

55 Consider the following about the Clean Development Mechanism (CDM).


1. It related to the Kyoto Protocol.
2. Only industrialized countries can participate in it.
3. Certified emission reduction (CER) credits are associated with CDM.
Select the correct answer using the codes below:
A. 1 and 2 only
B. 2 and 3 only
C. 1 and 3 only
D. 1, 2 and 3
User Answer :
Correct Answer : C
Answer Justification :

Learning: The Clean Development Mechanism (CDM), defined in Article 12 of the


Protocol, allows a country with an emission-reduction or emission-limitation
commitment under the Kyoto Protocol (Annex B Party) to implement an emissionreduction project in developing countries. Such projects can earn saleable certified
emission reduction (CER) credits, each equivalent to one tonne of CO2, which can
be counted towards meeting Kyoto targets.

IN

SI

A CDM project activity might involve, for example, a rural electrification


project using solar panels or the installation of more energy-efficient boilers.
The CDM allows industrialized countries to buy CERs and to invest in
emission reductions where it is cheapest globally. The CDM is designed to
start off developing countries on a path towards less pollution, with
industrialised (Annex B) countries paying for these reductions.

Q Source:
http://unfccc.int/kyoto_protocol/mechanisms/clean_development_mechanism/items/
2718.php

56 The MUDRA banks will be set up under the Pradhan Mantri MUDRA Yojana scheme.
It will
1. Provide its services to small entrepreneurs outside the service area of regular banks
2. Finance local commercial bank from the savings generated in the rural areas
3. Regulate micro-finance

(C) Insights Active Learning. | All rights reserved.

www.insightsias.com

39

Select the correct answer using the codes below:


A. 1 and 2 only
B. 2 and 3 only
C. 1 and 3 only
D. 1, 2 and 3
User Answer : C
Correct Answer : C
Answer Justification :

Sunanthaa

Total Marks

200

Mark Scored

84.67

IA
S

TEST - 7

User Name :

Learning:The MUDRA banks will be set up under the Pradhan Mantri MUDRA
Yojana scheme.

H
TS

It will provide its services to small entrepreneurs outside the service area of regular
banks, by using last mile agents (i.e. local last mile Financers). About 5.77 crore
(57.7 million) small business have been identified as target clients using the NSSO
survey of 2013 and Economic Survey 2014-15.

SI

Only 4% of these businesses get finance from regular banks (and rest from
exorbitant money lenders). The bank will also ensure that its clients do not
fall into indebtedness and will lend responsibly.
The bank will initially function as a non-banking financial company and a
subsidiary of the Small Industries Development Bank of India (SIDBI). Later,
it will be made into a separate company
It will also serve as a regulator for other micro-finance institutions (MFIs)
and provide them refinancing services. It will provide guidelines for MFIs
and give them ratings

IN

Eligible to borrow from MUDRA bank:


Small manufacturing unit
Shopkeepers
Fruits / Vegetable vendors
Artisans

Q Source:
http://www.thehindu.com/business/budget/mudra-bankto-fund-the-missing-middle/a
rticle6946797.ece

57 Which institution in India acts as the Debt Manager of the Government of India?
A. Reserve Bank of India
B. Life Insurance Corporation of India (LIC)

(C) Insights Active Learning. | All rights reserved.

www.insightsias.com

40

TEST - 7

User Name :

Sunanthaa

Total Marks :

200

Mark Scored :

84.67

C. Investment Management Company (IMCO)


D. Export - Import Bank of India (Exim)
User Answer : A
Correct Answer : A
Answer Justification :

IA
S

Justification: There was a recent proposal to set up a new Public Debt management
agency (PDMA). Before this, the jurisdiction of various institutions responsible for
public debt management is given below:

H
TS

I. Reserve Bank of India - Domestic Marketable Debt i.e., dated securities,


treasury bills and cash management bills.
II. Ministry of Finance (MOF); Office of Aid and accounts Division - external
debt
III. Ministry of Finance; Budget Division and Reserve Bank of India - Other
liabilities such as small savings, deposits, reserve funds etc.For monetary and
fiscal coordination, there is a cash and debt management committee which
meets regularly. The members comprise of officials from RBI and MOF.
Learning: In India, RBI is both Central banker and Debt management agency
(Treasury)

The proposal to setup a PDMA was based on some inherent Conflict of interests that
RBI faces:

IN

SI

By issuing bonds (in OMOs) at low interest rates, it can reduce government
debt, but since interest rates are low, few will buy bonds and excess liquidity
will cause inflation. On the other hand, interest rate should be raised to fight
inflation. Thus, there is a conflict.
Financial repression and borrowing - For e.g. RBI is the regulator of all bank
in India. Govt. raises debt from SLR obligations of banks, the limit for which
is mandated by RBI. SLR does not give high returns, but risk-free stable
returns. Since RBI is also the debt manager, there is a conflict of interest as
RBI may service govt. debt by forcing banks to hold higher SLR.
Inflation reduces the real debt of government. As RBI is the debt manager,
there is a conflict of interest, in reducing real debt of govt. and cutting down
in inflation.

Q
Source:http://www.thehindu.com/business/budget/public-debt-management-agency
-will-bring-indias-external-and-domestic-debt-under-one-roof-finance-minister-

(C) Insights Active Learning. | All rights reserved.

www.insightsias.com

41

TEST - 7

User Name :

Sunanthaa

Total Marks

200

Mark Scored

84.67

58 Writs can be issued by the High Courts to


1. Enforce a fundamental Right
2. Enforce a statutory Right
Which of the above is/are correct?
A. 1 only
B. 2 only
C. Both 1 and 2
D. None
User Answer : C
Correct Answer : C
Answer Justification :

IA
S

arun-jaitley/article6946273.ece

H
TS

Learning:A writ means an order. A warrant is also a type of writ. Anything that is
issued under an authority is a writ.
In this sense, using the power conferred by Article 32 and 226, the Supreme Court
and High courts issue directions, orders or writs.

Where SC can issue writs only for enforcing fundamental rights, HCs can also use it
to enforce a legal right, for e.g. right to property.
Q Source: Chapter 6: 11th NCERT: Indian Constitution

IN

SI

59 Floods destroy valuable crops every year and damage physical infrastructure. However,
floods are also helpful in which of the following way(s)?
1. Recharge of groundwater aquifiers
2. Deposition of fertile silt
3. Rejuvenation of the river ecosystem
Select the correct answer using the codes below:
A. 1 and 2 only
B. 2 and 3 only
C. 1 only
D. 1, 2 and 3
User Answer : D
Correct Answer : D
Answer Justification :
Justification:Statement 2 and 3: Every year, floods deposit fertile silt over

(C) Insights Active Learning. | All rights reserved.

www.insightsias.com

42

TEST - 7

User Name :

Sunanthaa

Total Marks :

200

Mark Scored :

84.67

agricultural fields which are good for the crops. Majuli (Assam), the largest riverine
island in the world, is the best example of good paddy crops after the annual floods
in Brahmaputra.
They also help in nutrient mixing of water, cleaning up wastes etc.

IA
S

Statement 1: Flood water also seeps to the ground and recharges the underground
aquifiers.
Q Source: Page 88: Chapter 7: 11th NCERT: India Physical Geography

SI

H
TS

60 Gold bond scheme allows investors to invest in sovereign bonds linked directly to price
of gold and will be paid off a market determined nominal interest rate. How will the
scheme help Indian economy?
1. Help reduce Current Account Deficit (CAD)
2. Allow government to channelize greater domestic savings to capital formation
3. Stabilize the price of Gold in international markets reducing vulnerability in India
Select the correct answer using the codes below:
A. 1 and 2 only
B. 2 and 3 only
C. 1 only
D. 2 only
User Answer : A
Correct Answer : A
Answer Justification :
Justification:Gold forms a large part of our imports. The scheme is likely to reduce
demand for physical gold. So, 1 is correct.

IN

Gold means unproductive idle savings. If the same is mobilized in formal financial
institutions, it can be used for infrastructural development. So, 2 is correct.
Statement 3 is far-fetched, as the scheme may oy may not stabilize international
prices of gold, which further depends on a variety of factors like global growth rate,
speculative forces etc.
Q Source:
http://www.thehindu.com/opinion/columns/the-sum-of-three-new-gold-schemes/arti
cle7869915.ece

(C) Insights Active Learning. | All rights reserved.

www.insightsias.com

43

TEST - 7

User Name :

Sunanthaa

Total Marks

200

Mark Scored

84.67

IA
S

61 The Human Development Index takes into account which of the following factors in its
calculation?
1. Rate of school drop outs
2. Sanitation and drinking water provisions
3. Adult literacy rate
Select the correct answer using the codes below:
A. 1 and 2 only
B. 2 and 3 only
C. 1 and 3 only
D. 3 only
User Answer : C
Correct Answer : D
Answer Justification :

H
TS

Learning: The Human Development Index (HDI) is a composite statistic of life


expectancy, education, and income per capita indicators, which are used to rank
countries into four tiers of human development.
A country scores higher HDI when the life expectancy at birth is longer, the
education period is longer, and the income per capita is higher. It includes:

Life Expectancy Index (LEI)


Education Index (EI) which includes Mean Years of Schooling Index (MYSI)
and Expected Years of Schooling Index (EYSI)
Income Index (II)

SI

The fact that HDI does not consider school dropout rates is considered a prime
drawback, which is very prevalent in developing countries like India. So, the years
of schooling would not reflect the education levels if the child has dropped out.

IN

Q Source: 2015 Human Development Index (HDI)

62 Which of the following fundamental rights is NOT available to linguistic minorities in


India?
A. Right to Inheritance of Ancestral property
B. Right to preservation of cultural heritage
C. Right to adopt, practice and propagate a religious belief
D. Right to freedom of association
User Answer : A
Correct Answer : A
Answer Justification :

(C) Insights Active Learning. | All rights reserved.

www.insightsias.com

44

TEST - 7

User Name :

Sunanthaa

Total Marks :

200

Mark Scored :

84.67

Justification: There is no fundamental right to inheritance. However, laws like


Hindu Succession Act regulate inheritance rights. So, (a) is wrong.
Rest are fundamental rights available under Article 19 (association) or 25-30
(culture and religion). So, (b), (c) and (d) is correct.

IA
S

A very important debate was going in the country on culture preservation rights in
the Niyamgiri case.
You can refer to these two links to understand the issue and the SC judgment.
http://www.downtoearth.org.in/coverage/niyamgiri-answers-41914

H
TS

http://www.thehindu.com/opinion/editorial/the-significance-of-niyamgiri/article467
7438.ece
Q Source: Chapter 2: 11th NCERT: Indian Constitution

SI

63 The festival of Losar has begun in Ladakh region of Jammu and Kashmir marks
beginning of
A. Harvest season in the Himalayan regions
B. New Year in this Himalayan region
C. Annual winter sports competitions across Himalayan regions
D. Joint mountaineering expedition in collaboration with Tibet
User Answer : A
Correct Answer : B
Answer Justification :

IN

Learning: It is a 10 day long festival began with illumination of religious and


residential places and by visiting monasteries for prayer.
Losar is Tibetan word for 'new year' and this festival is an important social and
religious celebration in Ladakh. Traditional Celebrations of festival
Following the old tradition, people from Ladakh visit graveyards of ancestor family
members and pray for peace for the departed soul.
People visit each other for exchange of Losar greetings.
On third day of the festival is people wait for the night to see the first moon of the
New Year In India, Losar is also celebrated by Yolmo, Sherpa, Tamang, Gurung,

(C) Insights Active Learning. | All rights reserved.

www.insightsias.com

45

TEST - 7

User Name :

Sunanthaa

Total Marks

200

Mark Scored

84.67

and Bhutia communities residing in different regions in the country.


Q Source:
http://www.thehindu.com/features/magazine/dragon-feast/article5538513.ece

H
TS

IA
S

64 Decisions of which of the following authorities may be binding on the Council of


Ministers?
1. Supreme Court and High Courts
2. Vice-President of India acting in his capacity
3. Union Public Service Commission
Select the correct answer using the codes below:
A. 1 and 3 only
B. 2 and 3 only
C. 1 only
D. 1, 2 and 3
User Answer : C
Correct Answer : C
Answer Justification :

Justification: Supreme Court and High Courts pass binding judgements, e.g. SR
Bommai Case, DC Wadhwa Case, issue writs etc. So, 1 is correct.

SI

Vice-President of India acting in his capacity does not have any de facto authority.
Only if he acts as the President, some of his decisions, for e.g. veto powers over bills
may be binding. So, 2 is incorrect.
Union Public Service Commission is an advisory body to the GoI. So, 3 is incorrect.

IN

Q Source: Chapter 4: 11th NCERT: Indian Constitution

65 The laissez-fairez theory aims to promote


A. Expansion of production with high productivity as a key objective
B. High exports and low imports
C. Free trade and free play of market forces
D. Development of a centrally planned economic system
User Answer :
Correct Answer : C
Answer Justification :
Learning: French for 'leave alone', laissez-faire is an economic theory that became

(C) Insights Active Learning. | All rights reserved.

www.insightsias.com

46

TEST - 7

User Name :

Sunanthaa

Total Marks :

200

Mark Scored :

84.67

popular in the 18th century. The driving idea behind laissez-faire as a theory was
that the less the government is involved in free market capitalism, the better off
business will be, and then by extension society as a whole.

IA
S

The underlying beliefs that make up the fundamentals of of laissez-faire economics


include first and foremost that the natural world is a self-regulating system, and that
natural regulation is the best type of regulation. Laissez-faire economists argue that
because of this there is no need for the complicating involvement of government.
Government involvement, according to this economic theory, would include any
type of regulation, minimum wage, taxation, or oversight. Laissez-faire economists
see taxation on companies as a penalty for production.

H
TS

Q Source: Chapter 1: Page 19: 11th NCERT: Understanding Society

SI

66 While the sun rises in the northeastern states about two hours earlier as compared to
Jaisalmer, the watches in Jaisalmer show the same time as in the NE states. Why?
1. We follow a standard time for the entire nation.
2. While NE states follow daylight saving, other states do not.
Which of the above is/are correct?
A. 1 only
B. 2 only
C. Both 1 and 2
D. None
User Answer : A
Correct Answer : A
Answer Justification :

IN

Justification: We do not follow daylight saving. So, 2 is wrong.


Daylight Saving Time (DST) is the practice of setting the clocks forward one hour
from Standard Time during the summer months, and back again in the fall, in order
to make better use of natural daylight.
You can go through this article to understand what the demand for introducing
daylight saving in India was.
http://www.thehindu.com/opinion/editorial/time-for-a-change/article5535448.ece
Q Source: Daylight saving was in news in 2014: Chapter 1: 11th NCERT: India
Physical Geography

(C) Insights Active Learning. | All rights reserved.

www.insightsias.com

47

TEST - 7

User Name :

Sunanthaa

Total Marks

200

Mark Scored

84.67

IA
S

67 What do you understand by the often used term "Cooperative Federalism"?


A. A healthy competition amongst the states of India for growth and
development
B. Constitutional division of powers between Centre and States
C. Centre and states working jointly to resolve issues of public interest
D. An ideal of federalism where states are given complete autonomy from the
Centre
User Answer : C
Correct Answer : C
Answer Justification :

Other elements are:

H
TS

Learning: Cooperative federalism is a concept of federalism in which national,


state, and local governments interact cooperatively and collectively to solve
common problems, rather than making policies separately.

Principle of subsidiarity - Responsibility for managing state/local specific


schemes/services etc should be shifted where it rightfully belongs, that is,
State and local levels.
Sharing of powers and responsibilities between the three levels of
government which involves participative policymaking.

Requisites for ensuring cooperative federalism are:

IN

SI

Greater devolution of financial resources towards the States.


Respect for the mandate of elected governments, even those run by
opposition parties.
These should be complemented by competitive federalism. States would vie
with each other to attract investments and provide better public goods and
services.

Q Source: The term appears often in news: Fourteenth Finance Commission


Reports

68 The "Principle of Subsidiarity" with respect to the relations between State governments
and Urban Local Bodies (ULBs) mean
A. The State governments should not try to control the ULBs.
B. The State governments should allow the ULBs to impose and collect taxes
C. The ULBs should work in cooperation with the other ULBs of the state even
outside their jurisdiction

(C) Insights Active Learning. | All rights reserved.

www.insightsias.com

48

TEST - 7

User Name :

Sunanthaa

Total Marks :

200

Mark Scored :

84.67

D. Responsibility and powers should be devolved to the level where it rightfully


belongs and is most efficient
User Answer : C
Correct Answer : D
Answer Justification :

IA
S

Justification: Principle of Subsidiarity is a 'core principle' of Local government as


per 2nd ARC.
It means what can be done best at the lower levels of government should not be
centalised at higher levels like cyber security; national defence etc. Its benefits are:

H
TS

Leads to efficiency, innovation, competition and self-reliance


Makes Indian democracy fully legitimate by making its truly devolved and
participatory
Citizenry will become more responsible and enlightened as they will
appreciate the hard choices that are to be made in governance. Leads to
maturing of democracy.
Q Source: The term appears often in news: 2nd ARC Local Government Report

IN

SI

69 Consider the following statements.


1. Some of the important fresh lakes such as Dal and Wular and salt water lakes such
as Pangong Tso and Tso Moriri are found in this region.
2. Some of the important passes of the region are Zoji La, Banihal, Photu La and
Khardung La.
The above refer to which region?
A. Northwestern Himalayas
B. Himachal and Uttaranchal Himalayas
C. Arunachal Himalayas
D. Eastern Hills and Mountains.
User Answer : B
Correct Answer : D
Answer Justification :
Learning: Kashmir or Northwestern Himalayas comprise a series of ranges such as
the Karakoram, Ladakh, Zaskar and Pir Panjal.
The northeastern part of the Kashmir Himalayas is a cold desert, which lies between
the Greater Himalayas and the Karakoram ranges.

(C) Insights Active Learning. | All rights reserved.

www.insightsias.com

49

TEST - 7

User Name :

Sunanthaa

Total Marks

200

Mark Scored

84.67

Between the Great Himalayas and the Pir Panjal range, lies the world famous valley
of Kashmir and the famous Dal Lake. Important glaciers of South Asia such as the
Baltoro and Siachen are also found in this region.
The Kashmir Himalayas are also famous for Karewa formations, which are useful
for the cultivation of Zafran, a local variety of saffron.

IA
S

Q Source: Page 11: Chapter 2: 11th NCERT: India Physical Geography

IN

SI

H
TS

70 The Pamir Knot falls to the North of which of the following in India?
1. Nanga Parbat
2. Shiwalik ranges
3. Kunlun Mountains
Select the correct answer using the codes below:
A. 1 and 2 only
B. 2 and 3 only
C. 1 only
D. 1, 2 and 3
User Answer : D
Correct Answer : A
Answer Justification :

Learning:
Q Source: Page 12: Chapter 2: 11th NCERT: India Physical Geography

(C) Insights Active Learning. | All rights reserved.

www.insightsias.com

50

TEST - 7

User Name :

Sunanthaa

Total Marks :

200

Mark Scored :

84.67

H
TS

IA
S

71 A Member of Legislative Assembly (MLA) has which of the following powers as given
by the constitution of India?
1. She can discretionally discontinue a development project initiated by the District
Administration falling in her constituency.
2. She can allocate finances between several divisions of the District.
3. She can get a bill introduced in the Legislative Assembly of the State which seeks to
regulate superstitious practices in the State.
Select the correct answer using the codes below:
A. 1 and 3 only
B. 2 and 3 only
C. 3 only
D. 1 and 2 only
User Answer : C
Correct Answer : C
Answer Justification :
Justification: Statement 1: No. An MLA does not have the discretion to discontinue
a development project. It can only be done by either the District government, or the
state government.
Statement 2: No. It falls in the purview of the state government.

Statement 3: A private member of the assembly can get a bill introduced in the
assembly.

SI

Q Source: Chapter 3: 11th NCERT: Indian Constitution

IN

72 Consider the following statements.


1. These hills are inhabited by the Lushais.
2. For the most part it is covered with dense bamboo jungle.
3. In the eastern portion of the hills open grass-covered slopes can be found.
4. Groves of oak and pine could be found interspersed with rhododendrons.
5. The Blue Mountain is the highest peak here.
The above refer to?
A. Garo Hills
B. Mizo Hills
C. Patkai Range
D. Jampui Hills
User Answer :
Correct Answer : B

(C) Insights Active Learning. | All rights reserved.

www.insightsias.com

51

TEST - 7

User Name :

Sunanthaa

Total Marks

200

Mark Scored

84.67

Answer Justification :

IA
S

Learning: The range is part of the Patkai range system The hills are for the most
part covered with dense bamboo jungle and rank undergrowth; but in the eastern
portion, owing probably to a smaller rainfall, open grass-covered slopes are found,
with groves of oak and pine interspersed with rhododendrons. The Blue Mountain is
the highest peak in Lushai hills.
These hills are inhabited by the Lushais and other Mizo tribes, but the population is
extremely scanty.
The management of the South Lushai hill country was transferred from Bengal to
Assam in 1898.

H
TS

Q Source: Page 15: Chapter 2: 11th NCERT: India Physical Geography

SI

73 Which of the following form a major water divide between the Arabian Sea Drainage
and Bay of Bengal Drainage in India?
A. Delhi Ridge
B. Aravalis
C. Sahyadris
D. All of the above
User Answer : A
Correct Answer : D
Answer Justification :
Learning: Indian drainage system may be divided on various bases. On the basis of
discharge of water (orientations to the sea), it may be grouped into:

IN

(i) the Arabian Sea drainage; and (ii) the Bay of Bengal drainage.
They are separated from each other through the Delhi ridge, the Aravalis and the
Sahyadris. Nearly 77 per cent of the drainage area consisting of the Ganga, the
Brahmaputra, the Mahanadi, the Krishna, etc. is oriented towards the Bay of Bengal
while 23 per cent comprising the Indus, the Narmada, the Tapi, the Mahi and the
Periyar systems discharge their waters in the Arabian Sea.
Q Source: Page 23: Chapter 3: 11th NCERT: India Physical Geography

74 Consider the following statements.

(C) Insights Active Learning. | All rights reserved.

www.insightsias.com

52

TEST - 7

User Name :

Sunanthaa

Total Marks :

200

Mark Scored :

84.67

IA
S

1. Western Ghats are higher in elevation than Eastern Ghats.


2. Western Ghats are more continuous than Eastern Ghats.
Which of the above is/are correct?
A. 1 only
B. 2 only
C. Both 1 and 2
D. None
User Answer : C
Correct Answer : C
Answer Justification :

H
TS

Learning: Western Ghats are comparatively higher in elevation and more


continuous than the Eastern Ghats. Their average elevation is about 1,500 m with
the height increasing from north to south.
'Anaimudi' (2,695 m), the highest peak of Peninsular plateau is located on the
Anaimalai hills of the Western Ghats followed by Dodabetta (2,637 m) on the
Nilgiri hills.

Eastern Ghats comprising the discontinuous and low hills are highly eroded by the
rivers such as the Mahanadi, the Godavari, the Krishna, the Kaveri, etc. Some of the
important ranges include the Javadi hills, the Palconda range, the Nallamala hills,
the Mahendragiri hills, etc. The Eastern and the Western Ghats meet each other at
the Nilgiri hills.

SI

Q Source: Page 17: Chapter 2: 11th NCERT: India Physical Geography

IN

75 The "Nitaqat Law" of Saudi Arabia was in news some time ago. It was related to
A. Deportation of unorganized Indian national labour from Saudi
B. Holding back property rights from Indian nationals living in Saudi
C. Debarring Indian nationals to hold public offices in Saudi
D. Give greater statutory preference to local Saudis in employment in firms
User Answer :
Correct Answer : D
Answer Justification :
Learning: In 2013, the government in Saudi decided to give more preference to
locals in jobs. For this purpose, it made the employment of 10% locals mandatory in
all firms. The Indians who were already working in these firms had to move out.
This created a big crisis as around 1 million workers were to be expatriated back to
India.

(C) Insights Active Learning. | All rights reserved.

www.insightsias.com

53

TEST - 7

User Name :

Sunanthaa

Total Marks

200

Mark Scored

84.67

The Indian government negotiated gradual implementation of the law so that the
workers get enough time to settle themselves. It also launched Pravasi bima yojana,
a scheme that helps them build their savings.

IA
S

Q Source:
http://indianexpress.com/article/world/middle-east-africa/nitaqat-law-to-punish-illeg
al-expatriates-saudi-arabia/

H
TS

76 As compared to the western coastal plain, the eastern coastal plain is broader and is an
example of an emergent coast. But, the eastern coastal plain has less number of ports and
harbours than the western side. Why?
1. The continental shelf extends far into the sea.
2. The eastern coastal plains are tectonically unstable.
3. The eastern coastal plains suffer from large inter-tidal variations that make it
difficult for ships to be stranded in the open sea.
Select the correct answer using the codes below:
A. 1 only
B. 2 and 3 only
C. 1 and 3 only
D. 1, 2 and 3
User Answer : C
Correct Answer : A
Answer Justification :

SI

Justification: Since the continental shelf extends far into the sea, the coastlines are
not very deep that can support standing cargo ships. So, 1 is correct.

IN

There are a number of ports on the eastern coasts. Therefore, whether tectonically
stable or unstable is not a reason for establishing ports. 2 is wrong.
Inter-tidal variations are actually good for ships as during high tide, it is easier for
ships to get stranded at the coast. So, 3 is incorrect.
Learning: Western submergence of Peninsula it is a narrow belt and provides
natural conditions for the development of ports and harbours. Kandla, Mazagaon,
JLN port Navha Sheva, Marmagao, Mangalore, Cochin, etc. are some of the
important natural ports located along the west coast.
Q Source: Page 18: Chapter 2: 11th NCERT: India Physical Geography

(C) Insights Active Learning. | All rights reserved.

www.insightsias.com

54

TEST - 7

User Name :

Sunanthaa

Total Marks :

200

Mark Scored :

84.67

H
TS

IA
S

77 Which of the following is/are the discretionary powers of the President of India?
1. The President can send back the advice given by the Council of Ministers and ask
the Council to reconsider the decision.
2. The President has veto power by which he can delay assent to Bills passed by the
Parliament.
3. Appointing Prime Minister based on his discretion even when a leader has a clear
majority in the Lok Sabha.
4. Dissolving the Lok Sabha when the government has lost confidence of the house
Select the correct answer using the codes below:
A. 1, 2 and 4 only
B. 2 and 3 only
C. 2 and 4 only
D. 1, 3 and 4 only
User Answer : A
Correct Answer : A
Answer Justification :
Justification: Statements 1 and 2 are correct as the President can exercise these
powers without the aid and advice of the council of ministers.
Statement 3 is incorrect as the President is not free to appoint anyone as the Pm, but
only the leader of the majority party/coalition.

SI

Statement 4 is correct as it is the President's obligation to maintain political order in


our Parliamentary democracy. To ensure this, he must dissolve the Lok Sabha when
it fails to represent the will of the people.
Q Source: Chapter 4: 11th NCERT: Indian Constitution

IN

78 Lai Autonomous District Council (LADC) is one of the three Autonomous District
Councils (ADCs) in Mizoram state in North-east India. An ADC is constituted under
A. Fifth Schedule of the Constitution
B. Sixth Schedule of the Constitution
C. Seventh Schedule of the Constitution
D. Ninth Schedule of the Constitution
User Answer : B
Correct Answer : B
Answer Justification :
Learning: The Sixth Schedule to the Constitution of India contains provisions
concerning the administration of tribal areas in the States of Assam, Meghalaya,

(C) Insights Active Learning. | All rights reserved.

www.insightsias.com

55

TEST - 7

User Name :

Sunanthaa

Total Marks

200

Mark Scored

84.67

Tripura and Mizoram.


It declares that the tribal areas in the North Cachar Hills District, the Karbi Anglong
District, the Bodoland Territorial Area District, the Khasi Hills District, the Jaintia
Hills District, the Garo Hills District and Tripura Tribal Areas District shall be an
autonomous district.

IA
S

In the upcoming tests, several other questions will be covered on this topic.
Q Source: Chapter 8: 11th NCERT: Indian Constitution

H
TS

79 What does the term "encryption" mean in the context of cyber security?
A. It is a process of encoding messages or information in such a way that only
authorized parties can read it.
B. It is the practice of restricting access to internet content based upon the user's
geographical location.
C. It relates to the exchange of information on a Virtual Private Network (VPN).
D. None of the above
User Answer : A
Correct Answer : A
Answer Justification :

Learning: Encryption is the process of encoding messages or information in such a


way that only authorized parties can read it.

SI

For example: word "ABC" can become "BCD" in encrypted form, if every letter is
replaced by next alphabet. Those who know how it is encoded can only able to read
"ABC" correctly.

IN

All messaging services like WhatsApp, Viber, Google Chat, Yahoo messenger use
encrypted services. Banks (payment gateways) and e-commerce sites also use
encryption to protect financial and private data including passwords.
Under Section 84A of Information Technology Act, 2000 Rules are to be framed to
prescribe modes or methods for encryption
The Government of India recently published the Draft National Encryption policy
for public comments
Q Source: In news due to Draft National Encryption Policy release

(C) Insights Active Learning. | All rights reserved.

www.insightsias.com

56

TEST - 7

User Name :

Sunanthaa

Total Marks :

200

Mark Scored :

84.67

IA
S

80 Which of the following is most appropriately related to the often used term "Judicial
Activism"?
A. When the Judiciary declares that it wields executive and legislative powers
B. When the Judiciary declares void an executive order that impinges on public
interest
C. When the Judiciary exercises self-restraint principle and is passive in
interpretation of laws
D. When the Judiciary disregards Parliamentary laws and executive rules
User Answer : A
Correct Answer : B
Answer Justification :

H
TS

Learning: Judicial Activism is one step ahead of judicial review - an extension of


judicial review. It is when the Judiciary takes upon those issues that are normally
left to be dealt by the legislature or the executive for e.g. the judgement on
transgenders, on police reforms in 2006. The personal views of the judges, rather
than existing laws, may also guide the decisions.
It is a departure of the Judiciary from the self-restraint principle and passive
interpretation of law to an active social engineer and law upholder. It stems from the
broader interpretation of existing statues and FRs.

SI

The Supreme Court has ordered to set up a special 'Social Justice Bench' to
exclusively hear cases concerning social issues by way of PIL. The issues would
range from women safety to mid-day meals in schools and night shelters for the
under privileged.
Q Source: Frequently in news and important for Prelims.

IN

81 Which of the following differentiates the evolution of the Himalayan Drainage pattern
from that of the Peninsular?
1. Peninsular river system is older than the Himalayan river system.
2. Upheaval of Himalayas affected the Himalayan river system, but not the peninsular
river system.
3. Peninsular river system was deformed due to subsidence of the western flank of the
Indian Peninsula, but this developed Himalayan Rivers.
Select the correct answer using the codes below:
A. 1 and 2 only
B. 1 only
C. 2 and 3 only

(C) Insights Active Learning. | All rights reserved.

www.insightsias.com

57

TEST - 7

User Name :

Sunanthaa

Total Marks

200

Mark Scored

84.67

D. 1, 2 and 3
User Answer : A
Correct Answer : B
Answer Justification :

IA
S

Learning & Justification: These points explain the differences between both in
order:

SI

H
TS

A. Himalayan river system is believed to have originated from a mighty river


called Indo-Brahma in the Miocene period.
Peninsular system is older than the Himalayan river
B. Himalayan is dismembered in three groups - Indus system, Ganges and
Brahmaputra and tributaries of each.
Peninsular did not dismember, but subsidence of the western flank of
the Peninsula leading to its submergence below the sea disturbed the
symmetrical plan of the river system.
C. Dismemberment between Indus system and Ganges was caused by upheaval
in western Himalayas (Pleistocene age) and upliftment of Potwar plateau
(Delhi ridge).
Upheaval of Himalayas led to trough faulting (and not
dismemberment) in Peninsula. Today Narmada and Tapti flow in the
trough faults.
D. Down-thrusting of the Malda gap area between the Rajmahal hills and the
Meghalaya plateau (Pleistocene period), diverted the Ganges and the
Brahmaputra systems to flow towards the Bay of Bengal.
This down-thrusting did not affect the flow of Peninsular rivers
towards Bay of Bengal. But, slight tilting of the Peninsular block from
northwest to the South-eastern direction gave orientation to the entire
drainage system towards the Bay of Bengal.

IN

82 The Indus receives a number of Himalayan tributaries which include


1. Gilgit
2. Nubra
3. Zaskar
4. Shigar
Select the correct answer using the codes below:
A. 1 and 2 only
B. 1 and 4 only
C. 2 and 3 only
D. 1, 2, 3 and 4
User Answer :

(C) Insights Active Learning. | All rights reserved.

www.insightsias.com

58

TEST - 7

User Name :

Sunanthaa

Total Marks :

200

Mark Scored :

84.67

Correct Answer : D
Answer Justification :

IA
S

Learning: The Indus receives a number of Himalayan tributaries such as the Shyok,
the Gilgit, the Zaskar, the Hunza, the Nubra, the Shigar, the Gasting and the Dras. It
finally emerges out of the hills near Attock where it receives the Kabul river on its
right bank.
The other important tributaries joining the right bank of the Indus are the Khurram,
the Tochi, the Gomal, the Viboa and the Sangar. They all originate in the Sulaiman
ranges. The river flows southward and receives 'Panjnad' a little above Mithankot.

H
TS

The Panjnad is the name given to the five rivers of Punjab, namely the Satluj, the
Beas, the Ravi, the Chenab and the Jhelum. It finally discharges into the Arabian
Sea, east of Karachi. The Indus flows in India only through the Leh district in
Jammu and Kashmir.
Q Source: Page 25: Chapter 3: 11th NCERT: India Physical Geography

SI

83 The elections to the Rajya Sabha is based on


A. Mixed Member Proportional system
B. Alternative Vote/Instant-runoff voting
C. Majority Bonus System
D. Single Transferable Vote system
User Answer : A
Correct Answer : D
Answer Justification :

IN

Learning: The Single Transferable Vote system (STV) is a variant of the PR


system.
It is followed for Rajya Sabha elections. Every State has a specific quota of seats in
the Rajya Sabha. The members are elected by the respective State legislative
assemblies. The voters are the MLAs in that State.
Every voter is required to rank candidates according to her or his preference. To be
declared the winner, a candidate must secure a minimum quota of votes
Q Source: Page 57: Chapter 3: 11th NCERT: Indian Constitution

(C) Insights Active Learning. | All rights reserved.

www.insightsias.com

59

TEST - 7

User Name :

Sunanthaa

Total Marks

200

Mark Scored

84.67

IA
S

84 The Brahmaputra is well-known for floods, channel shifting and bank erosion. Which of
the following can explain it?
1. Brahmaputra's river bed has zero seepage capacity.
2. Large quantity of sediments brought by its tributaries increase the water level.
Which of the above is/are correct?
A. 1 only
B. 2 only
C. Both 1 and 2
D. None
User Answer : B
Correct Answer : B
Answer Justification :

H
TS

Justification: This is due to the fact that most of its tributaries are large, and bring
large quantity of sediments owing to heavy rainfall in its catchment area.
This increases the water level and cause heavy floods in regions like Assam. So, 2 is
correct.
A river bed will always have some seepage capacity, meaning that water will seep
down from the river bed to groundwater in case of excess discharge. It helps control
flood water. So, 1 is incorrect.

Q Source: Page 26: Chapter 3: 11th NCERT: India Physical Geography

IN

SI

85 The President can be removed from office by a process of impeachment for


A. Not following the advice of the Vice-President
B. Violation of the Constitution
C. Approving decisions that impinge on public interest
D. Speaking against the Council of Ministers
User Answer : B
Correct Answer : B
Answer Justification :
Justification: The President can be removed from office only by Parliament by
following the procedure for impeachment.
This procedure requires a special majority as explained in the last chapter.
The only ground for impeachment is violation of the Constitution. However, the
phrase has not been defined in the constitution.

(C) Insights Active Learning. | All rights reserved.

www.insightsias.com

60

TEST - 7

User Name :

Sunanthaa

Total Marks :

200

Mark Scored :

84.67

For e.g. if the council of ministers tenders an unconstitutional advice to President


and if he signs it, whether it is violation of the constitution or not? This is because
not following the advice of the CoM is also a violation of the constitution.
Q Source: page 84: Chapter 4: 11th NCERT: Indian Constitution

H
TS

IA
S

86 There is a lack of alluvial and deltaic deposits in Narmada and Tapi rivers because
A. They do not erode the river bed.
B. They flow in trough faults and fill the original cracks with their detritus
materials.
C. Their sediments are ploughed back by distributaries.
D. They make large estuaries and lagoons away from the coast.
User Answer : B
Correct Answer : B
Answer Justification :
Justification: The Narmada and The Tapi flow in trough faults and fill the original
cracks with their detritus materials. Hence, there is a lack of alluvial and deltaic
deposits in these rivers. So, (b) is correct.

The Damodar, which occupies the eastern margins of the Chotanagpur Plateau, also
flows through a rift valley.
Q Source: Page 27: Chapter 3: 11th NCERT: India Physical Geography

IN

SI

87 The physiography or relief of India affects which of the following climatic variables?
1. Temperature
2. Air pressure
3. Direction and speed of wind
4. Amount and distribution of rainfall
Which of the above is/are correct?
A. 1 and 4 only
B. 2 only
C. 1, 3 and 4
D. 1, 2, 3 and 4
User Answer : D
Correct Answer : D
Answer Justification :
Justification: Himalayas is one classic example. So, 1 would be correct.

(C) Insights Active Learning. | All rights reserved.

www.insightsias.com

61

TEST - 7

User Name :

Sunanthaa

Total Marks

200

Mark Scored

84.67

Mountainous areas have low air pressure. So, 2 is correct.


Mountains like Western Ghats change the direction and speed of moisture laden
winds. So, 3 is correct.

IA
S

The windward sides of Western Ghats and Assam receive high rainfall than the
adjoining leeward sides. So, 4 is also correct.
Q Source: Page 35: Chapter 4: 11th NCERT: India Physical Geography

H
TS

88 Consider the following about the mechanism of Weather changes in the Summer Season
in India.
1. The Inter Tropical Convergence Zone (ITCZ) shifts southwards.
2. Westerly jet stream withdraws from the Indian region.
Which of the above is/are correct?
A. 1 only
B. 2 only
C. Both 1 and 2
D. None
User Answer :
Correct Answer : B
Answer Justification :

SI

Justification: As the summer sets in and the sun shifts northwards, the wind
circulation over the subcontinent undergoes a complete reversal at both, the lower as
well as the upper levels.

IN

By the middle of July, the low pressure belt nearer the surface [termed as Inter
Tropical Convergence Zone (ITCZ)] shifts northwards, roughly parallel to the
Himalayas between 20 N and 25 N with the apparent movement of the Sun. So, 1
is incorrect.
Also, by this time, the westerly jet stream withdraws from the Indian region. So, 2 is
correct.
Learning: In fact, meteorologists have found an interrelationship between the
northward shift of the equatorial trough (ITCZ) and the withdrawal of the westerly
jet stream from over the North Indian Plain. It is generally believed that there is a
cause and effect relationship between the two.
The ITCZ being a zone of low pressure, attracts inflow of winds from different

(C) Insights Active Learning. | All rights reserved.

www.insightsias.com

62

TEST - 7

User Name :

Sunanthaa

Total Marks :

200

Mark Scored :

84.67

directions. The maritime tropical airmass (mT) from the southern hemisphere, after
crossing the equator, rushes to the low pressure area in the general southwesterly
direction. It is this moist air current which is popularly known as the southwest
monsoon.

IA
S

Q Source: Page 36: Chapter 4: 11th NCERT: India Physical Geography

H
TS

89 Consider the following about the Law Commission of India.


1. It is a statutory body.
2. It acts as the legal representative of the Government in court hearings
3. The Law Secretary to the Union Government is the ex-officio member of the
Commission.
Select the correct answer using the codes below:
A. 1 and 3 only
B. 2 and 3 only
C. 1 only
D. 1 and 2 only
User Answer :
Correct Answer : C
Answer Justification :

Justification: Law commission of India is neither a constitutional nor a statutory


body. Rather, it was established by an executive resolution of the GoI to make
recommendations for law reforms. So, 1 would be wrong.

IN

SI

The Commission is established for a fixed tenure (generally three years) and
consists of a Chairperson and four other full-time members. The Law Secy. and the
Secy Legislative Department are the ex-officio members of the Commission. So, 3
would be correct.
It is the Attorney general or the Solicitor generals who represent Indian government
in court hearings. So, 2 is wrong.
Q Source:
http://indianexpress.com/article/india/india-others/law-commission-recommends-ab
olition-of-death-penalty/

90 During the south-west monsoon period after heavy rains for a few days, if rain fails to
occur for one or more weeks, it is known as break in the monsoon. These breaks in the
different regions can be due to

(C) Insights Active Learning. | All rights reserved.

www.insightsias.com

63

TEST - 7

User Name :

Sunanthaa

Total Marks

200

Mark Scored

84.67

IA
S

1. Wind blowing parallel to the coast in Western coastal areas.


2. The appearance of the Western Tropical Cyclones from the Arabian Sea
Which of the above is/are correct?
A. 1 only
B. 2 only
C. Both 1 and 2
D. None
User Answer : A
Correct Answer : A
Answer Justification :

H
TS

Learning & Justification: These breaks in rainfall are related to the cyclonic
depressions mainly formed at the head of the Bay of Bengal, and their crossing into
the mainland. Besides the frequency and intensity of these depressions, the passage
followed by them determines the spatial distribution of rainfall.
Moreover, these breaks in the different regions are due to different reasons:
In northern India rains are likely to fail if the rain-bearing storms are not very
frequent along the monsoon trough or the ITCZ over this region.
Over the west coast the dry spells are associated with days when winds blow
parallel to the coast. So, 1 is correct.

2 is incorrect as Western Tropical Cyclones originate from the Mediterranean Sea,


and they usually withdraw with the onset of the Monsoon.

SI

Q Source: Page 49: Chapter 4: 11th NCERT: India Physical Geography

IN

91 The "Leader of opposition" in either house of Parliament is


1. The Leader of all the Parliamentary Committees
2. Appointed by the Speaker of the House
Which of the above is/are correct?
A. 1 only
B. 2 only
C. Both 1 and 2
D. None
User Answer : B
Correct Answer : D
Answer Justification :
Learning: About Leader of Opposition

(C) Insights Active Learning. | All rights reserved.

www.insightsias.com

64

TEST - 7

User Name :

Sunanthaa

Total Marks :

200

Mark Scored :

84.67

IA
S

A politician who leads the opposition in either house of Parliament.


The term 'leader of opposition' is defined in Salary and Allowances of
Leaders of Opposition in Parliament Act, 1977.
Is the leader of that House of the Party in Opposition to the Government
having the greatest numerical strength and recognized, as such, by
the Chairman of Rajya Sabha or the Speaker of Loksabha.
Formal Recognition by a convention set by G.V Mavalakar- Party seeking
opposition role must have atleast 10% of the total strength of the House.
Criterion is 10% for single party not an alliance (pre and post election)
Though the statutory and legal status is given by the above mentioned act but
decision is taken by Speaker of LokSabha or Chairman of RajyaSabha, as the
case may be.

H
TS

Q Source: Controversy surrounding LoP last year 2014

http://www.thehindu.com/news/national/congress-entitled-to-leader-of-opposition-p
ost-in-lok-sabha-sonia-gandhi/article6185724.ece
http://indianexpress.com/article/opinion/columns/opposition-without-a-head/

IN

SI

92 Regulatory control over the quality, safety and efficacy of drugs in the country is
exercised through
A. Drugs and Cosmetics Act, 1940
B. The Pharmacy Act, 1948
C. Indian Medical Council Act, 1956
D. Indian Red Cross Society Act, 1920
User Answer : A
Correct Answer : A
Answer Justification :
Learning: The Drugs and Cosmetics Act, 1940 is a Central Act implemented
mainly through the States / UTs which have the major responsibilities to grant /
renew the drugs manufacturing licenses and sale licenses and enforcement of
various provisions of Drugs and Cosmetics Act and Rules including.
At present, the drug regulatory authorities of the States / UT Governments have
grossly inadequate infrastructure and manpower.
Considering the sensitivity and importance of the Pharmaceutical Sector and lack of
resources available with State / UT Governments, a Centrally Sponsored Scheme to
strengthen their infrastructure, both physical and human resources, has been

(C) Insights Active Learning. | All rights reserved.

www.insightsias.com

65

TEST - 7

User Name :

Sunanthaa

Total Marks

200

Mark Scored

84.67

envisaged during the 12th Five Year Plan.


Q Source:
http://www.mohfw.nic.in/index1.php?lang=1&level=5&sublinkid=4475&lid=2153

H
TS

IA
S

93 Consider the following statements.


1. Tropical cyclones only occur during Monsoon season in India.
2. Tropical cyclones occur more in Arabian Sea than Bay of Bengal Sea.
Which of the above is/are correct?
A. 1 only
B. 2 only
C. Both 1 and 2
D. None
User Answer : A
Correct Answer : D
Answer Justification :

Justification: In the table below (taken from the Q Source), you can see that
cyclones occur in nearly every month - more pronounced in Oct-Nov time.
The reason is that conditions exist throughout the months. So, 1 is incorrect.

IN

SI

Tropical cyclones occur more in the Bay of Bengal Sea rather than Arabian Sea as
more favourable conditions exist there in form of higher temperatures, presence of
easterlies etc. So, 2 is incorrect.

(C) Insights Active Learning. | All rights reserved.

www.insightsias.com

66

TEST - 7

User Name :

Sunanthaa

Total Marks :

200

Mark Scored :

84.67

Q Source: Page 86: Chapter 7: 11th NCERT: India Physical Geography

IA
S

94 What can possibly happen if the sun shines vertically above the Tropic of Capricorn in
the southern hemisphere?
A. Low pressure develops over North-western India due to high temperatures.
B. The ITCZ shifts north-westwards.
C. It increases the probability of occurrence of El-nino and La-nina in India.
D. High pressure develops over North-western India due to low temperatures.
User Answer : D
Correct Answer : D
Answer Justification :

H
TS

Justification: By the end of December, the sun shines vertically over the Tropic of
Capricorn in the southern hemisphere.
The weather in this season is characterised by feeble high pressure conditions over
the northern plain. In south India, the air pressure is slightly lower.
As a result, winds start blowing from north-western high pressure zone to the low air
pressure zone over the Indian Ocean in the south.

Hence, (d) is correct.

Q Source: Page 42: Chapter 4: 11th NCERT: India Physical Geography

IN

SI

95 In India, wetlands can be found in


1. Gulf of Kachchh
2. Lagoons of India's east coast
3. Floodplains of the Brahmaputra
Select the correct answer using the codes below:
A. 1 and 2 only
B. 2 and 3 only
C. 1 only
D. 1, 2 and 3
User Answer : D
Correct Answer : D
Answer Justification :
Learning: The country's wetlands have been grouped into eight categories, viz.

(C) Insights Active Learning. | All rights reserved.

www.insightsias.com

67

TEST - 7

User Name :

Sunanthaa

Total Marks

200

Mark Scored

84.67

IA
S

i. the reservoirs of the Deccan Plateau in the south together with the lagoons
and other wetlands of the southern west coast;
ii. the vast saline expanses of Rajasthan, Gujarat and the Gulf of Kachchh; (iii)
freshwater lakes and reservoirs from Gujarat eastwards through Rajasthan
(Keoladeo National Park) and Madhya Pradesh;
iii. the delta wetlands and lagoons of India's east coast (Chilika Lake); (v) the
freshwater marshes of the Gangetic Plain;
iv. the floodplains of the Brahmaputra; the marshes and swamps in the hills of
northeast India and the Himalayan foothills;
v. the lakes and rivers of the montane region of Kashmir and Ladakh; and
vi. the mangrove forest and other wetlands of the island arcs of the Andaman and
Nicobar Islands.

H
TS

Q Source: Chapter 5: Page 60: 11th NCERT: India Physical Geography

SI

96 UNESCO's Man and Biosphere (MAB) Programme aims to


A. Establish a scientific basis for the improvement of relationships between
people and their environments
B. Implement voluntary action plans of the United Nations with regard to
sustainable development.
C. Adaptation of international products around the particularities of a local
human culture
D. Identify human heritages and then support them financially and technically
User Answer : B
Correct Answer : A
Answer Justification :

IN

Learning: The MAB programme provides a unique platform for cooperation on


research and development, capacity-building and networking to share information,
knowledge and experience on three interlinked issues: biodiversity loss, climate
change and sustainable development.
It contributes not only to better understanding of the environment, but also promotes
greater involvement of science and scientists in policy development concerning the
wise use of biological diversity.
To date, 651 biosphere reserves in 120 countries, including 15 transboundary sites,
have been included in the World Network of Biosphere Reserves
Q Source: Chapter 5: Page 63: 11th NCERT: India Physical Geography

(C) Insights Active Learning. | All rights reserved.

www.insightsias.com

68

TEST - 7

User Name :

Sunanthaa

Total Marks :

200

Mark Scored :

84.67

IA
S

97 The "Operation Raahat" undertaken by the Government recently relates to


A. Flushing out of Bodo militants from Assam by the Indian armed Forces
B. Operation of the Indian Armed Forces to evacuate Indian citizens and other
foreign nationals from Yemen Crisis
C. India's Army-led rescue and relief mission in quake-hit Nepal
D. A cross-border counter-insurgency raid in Myanmar
User Answer : B
Correct Answer : B
Answer Justification :
Justification: Operation All Out (2015) - for flushing out Bodo militants Assam
Operation Maitri (2015) -India's Army-led rescue and relief mission in quake-hit
Nepal

H
TS

2015 Indian counter-insurgency operation in Myanmar - A cross-border counterinsurgency raid in Myanmar.


Learning: In December 2014, a series of attacks by militants resulted in deaths of
more than 76 in India. The attacks took place in Chirang, Sonitpur and Kokrajhar
districts, in December 2014. They have been attributed to the Songbijit faction of
National Democratic Front of Bodoland NDFB(S).

SI

On 26th December, the Government of India declared the launch of Operation All
Out to eliminate the Bodo militants and deployed soldiers of the Indian Army and
the Central Reserve Police Force.

IN

Q Source:
http://www.thehindu.com/specials/the-great-yemen-escape-operation-rahat-by-numb
ers/article7089422.ece

98 If you visit the state of Jammu & Kashmir, which soil(s) you are most likely to find?
1. Red Soil
2. Laterite Soil
3. Forest Soils
4. Black Soil
Select the correct answer using the codes below:
A. 1 and 2 only
B. 2 and 4 only
C. 3 only
D. 1 and 3 only

(C) Insights Active Learning. | All rights reserved.

www.insightsias.com

69

TEST - 7

User Name :

Sunanthaa

Total Marks

200

Mark Scored

84.67

User Answer : C
Correct Answer : C
Answer Justification :

IA
S

Learning: In Kathua and Jammu mainly alluvial soils are found, which are loamy
with little clay content and contain small quantity of lime with high magnesium
content.
There are three parallel belts widely apart from Forest and Hill soils, one stretching
from Poonch to Kathua in Jammu province second North West of Jhelum valley in
Kashmir province and the third belt stretching from south eastern part of Ladakh
range. The soils are generally mixed with pebbles.

H
TS

In southern part of Udhampur and Doda district brown soil under Deciduous Forest
are found. Colour of the soil is dark-brown and varies from dry loams to silt loams
with gravels in a small percentage.
In middle Ladakh range two isolated patches (one in Ladakh and another in Doda
district) of Podzolised soil occur over a long stretch.
In Poonch, Udhampur and Anantnag district sub-mountain soils are mainly found. In
the valley this soil is cultivated intensively and rice is the main crop.

Q Source: Chapter 6: 11th NCERT: India Physical Geography

IN

SI

99 "Connect Central Asia policy" (2012) of India deals with which of the following?
1. Transportation between India and Central Asia
2. Partnership in the Development of energy and natural resources
3. Development of potential in medical field
4. Scrapping of the visa regimes for select Central Asian Countries in India
Select the correct answer using the codes below:
A. 1 only
B. 2, 3 and 4 only
C. 1, 2 and 3 only
D. All of the above
User Answer : D
Correct Answer : C
Answer Justification :
Learning: Some of the elements of India's 'Connect Central Asia' policy as follow:

(C) Insights Active Learning. | All rights reserved.

www.insightsias.com

70

TEST - 7

User Name :

Sunanthaa

Total Marks :

200

Mark Scored :

84.67

IN

SI

H
TS

IA
S

India will strengthen its strategic and security cooperation. India already has
strategic partnerships in place with some Central Asian countries. In focus
will be military training, joint research, counter-terrorism coordination and
close consultations on Afghanistan.
India looks to Central Asia as a long term partner in energy, and natural
resources. Central Asia possesses large cultivable tracts of land and it sees
potential for India to cooperate in production of profitable crops with value
addition.
The medical field is another area that offers huge potential for cooperation.
India is ready to extend cooperation by setting up civil hospitals/clinics in
Central Asia.
India would like to assist in the setting up of a Central Asian University in
Bishkek that could come up as a centre of excellence to impart world class
education in areas like Information Technology, management, philosophy
and languages.
India is working on setting up a Central Asian e-network with its hub in
India, to deliver, tele-education and tele-medicine connectivity, linking all the
five Central Asian States.
Indian companies can showcase its capability in the construction sector and
build world class structures at competitive rates. Central Asian countries,
especially Kazakhstan, have almost limitless reserves of iron ore and coal, as
well as abundant cheap electricity. India can help set up several medium size
steel rolling mills, producing its requirement of specific products.
As for land connectivity, India has reactivated the International North-South
Transport Corridor (INSTC). India & Central Asian nations need to join our
efforts to discuss ways to bridge the missing links in the Corridor at the
earliest and also work on other connecting spurs along the route.
Absence of a viable banking infrastructure in the region is a major barrier to
trade and investment. Indian banks can expand their presence if they see a
favourable policy environment.

Q Source:
http://www.mea.gov.in/Speeches-Statements.htm?dtl/19791/Keynote+address+by+
MOS+Shri+E+Ahamed+at+First+IndiaCentral+Asia+Dialogue
http://archive.indianexpress.com/news/crossborder-terror-on-agenda-at-talks-betwee
n-indiatajikistan/1103128/

100 KISAN Project launched by Ministry of Agriculture & Farmers Welfare intends to
A. Use Space Technology and geo-informatics to improve yield estimation in

(C) Insights Active Learning. | All rights reserved.

www.insightsias.com

71

TEST - 7

User Name :

Sunanthaa

Total Marks

200

Mark Scored

84.67

IA
S

farms
B. Strengthening of fertilizer and soil nutrient quality control laboratories in
states
C. Provide single window approach and investment friendly atmosphere for
agricultural marketing in India
D. Meet the requirements of farmers for storing farm produce, consumer articles
and agricultural inputs
User Answer : A
Correct Answer : A
Answer Justification :

H
TS

Learning: The project envisages use of Space Technology and geoinformatics (GIS,
GPS and Smartphone) technology along with high resolution data from UAV/Drone
based imaging for improvement in yield estimation and better planning of Crop
Cutting Experiments (CCEs), needed for crop insurance programme.
The Pilot Study is proposed to be launched in one District each of Haryana,
Karnataka, Madhya Pradesh and Maharashtra during Kharif season of 2015 and two
Districts each of these States during Rabi season

IN

SI

Q Source: http://pib.nic.in/newsite/PrintRelease.aspx?relid=128429

(C) Insights Active Learning. | All rights reserved.

www.insightsias.com

72

Você também pode gostar